AGACNP Spring 2024 Exam 2

Lakukan tugas rumah & ujian kamu dengan baik sekarang menggunakan Quizwiz!

A 45-year-old man with a history of Hashimoto thyroiditis presents to the emergency department with new onset jaundice. He is easily fatigued and breathless on exertion. His pulse rate is 120 bpm, and his hematocrit is 16%. The patient states that he has never been transfused. Blood bank evaluation indicates that the patient has a serum antibody that is reactive with all reagent red cells and that his DAT is positive with anti-IgG but negative with anti-C3d reagents. What is the most appropriate management? A. Have blood bank continue crossmatching until compatible units are identified B. Infuse crystalloid or colloid because crossmatch-compatible blood is not available C. Send a new blood sample for compatibility testing D. Transfuse the crossmatch-incompatible blood

2. The correct answer is D. Rationale: The patient in this question has severe, symptomatic hemolytic anemia and is at high risk for ischemic tissue injury. The transfusion of crossmatch-incompatible blood is warranted (choice D is correct). There is no convincing evidence that transfused red cells are hemolyzed more quickly than the patient's own red cells unless the patient has underlying blood group alloantibodies. It is highly unlikely that a male patient who has never been transfused would have alloantibodies that could put him at risk for hemolysis. He will likely not benefit sufficiently from either crystalloid or colloid infusion alone (choice B is incorrect). And repeat testing for compatibility is unlikely to work and will just delay therapy (choices A and C are incorrect).

A 25-year-old woman with a history of epilepsy, depression, and seasonal allergies undergoes a urine drug screen for work clearance which returns positive for phencyclidine (PCP). The patient adamantly denies using illicit drugs. Which of her medications most likely caused the false-positive urine drug screen? A. Diphenhydramine B. Lamotrigine C. Venlafaxine D. All of the above

The correct answer is D. Rationale: All of the medications listed may cause false-positive results for phencyclidine on urine drug screens. Although not listed here, another common over-the-counter agent that does this is dextromethorphan.

Which one of the following is most correlated with the severity of calcium channel blocker (CCB) poisoning? A. Blood glucose B. Blood pressure C. Heart rate D. Temperature

The correct answer is A. Rationale: Blood glucose, as compared with vital signs, appears to best correlate with the severity of calcium channel blocker poisoning (choice A is correct). Of note, dihydropyridines maintain normal sinus rhythm and can cause reflex sinus tachycardia.

Which of the following metabolites is measured when screening a patient's urine for cocaine use? A. Benzoylecgonine B. Ecgonine methyl ester C. Hydroxycocaine D. Norcocaine

The correct answer is A. Rationale: The standard assay used to detect cocaine is an immunoassay for the metabolite benzoylecgonine, which has extremely high specificity, meaning false-positive results are extremely rare (choice A is correct). Coca leaves or coca tea, which is illegal in the United States, and, rarely, dental procedures in which cocaine is used for topical anesthesia, will show up positive on this test.

Which of the following statements is true regarding annual use of ethanol in the United States? A. It contributes to an estimated 100,000 deaths B. It is a factor in up to 8% of hospital admissions C. It is moderately to heavily consumed by 20% of the population D. It has an economic impact of 50 billion dollars

The correct answer is A. Rationale: Ethanol is estimated to contribute to nearly 100,000 deaths (choice A is correct), 3 million years of potential life lost, and economic costs more than $250 billion per year in the United States. It is a factor in 8% of emergency department visits and up to 50% of hospital admissions. Most Americans use alcohol, with about a third being moderate to heavy drinkers, as defined as four or more drinks per week.

Which one of the following best characterizes the opioid toxidrome? A. Miosis, bradypnea, bronchorrhea, profound diaphoresis, seizures, and depressed mental status B. Miosis, sluggish speech, confused or sleepy, respiratory depression, and feeling cold C. Mydriasis, diaphoresis, agitation, unable to sit still, and picking at skin instead of engaging in a conversation D. Mydriasis, dry mucous membranes, hot and flushed skin, and hallucinations of bugs crawling

1. The correct answer is B. Rationale: The opioid toxidrome is characterized by miosis, ventilatory and central nervous system depression, and hypothermia (choice B is correct). Choice A would be consistent with cholinergic toxidrome. Choice C is the sympathomimetic toxidrome, while choice D is consistent with anticholinergic toxidrome.

An 81-year-old man is critically ill with gram-negative sepsis. He is on vasopressors, broad-spectrum antibiotics, and every other day hemodialysis. He is noted to have spontaneous bleeding from his central line and hemodialysis catheter. Laboratory findings are as follows: Hemoglobin 7 g/dL (12-16 g/dL), WBC 45,000/mm3 (4000-11,000/mm3), platelets 23,000/mm3 (150- 400,000/mm3), PT 30 s (9.5-11.7 s), PTT 60 s (23-32 s), INR 3, and fibrinogen 50 mg/dL (150-450 mg/dL). What is the most appropriate next step in management? A. Heparin drip B. More frequent hemodialysis C. Transfuse 4 units of FFP D. Transfuse cryoprecipitate

The correct answer is D. Rationale: This presentation is suggestive of disseminated intravascular coagulation (DIC). Treatment of DIC includes treating the underlying disorder and replacing consumed factors. Here, the next step would be to replace fibrinogen by transfusing cryoprecipitate to a level >100 mg/dL (choice D is correct). Human clinical trials have reported mixed results with the use of heparin to treat DIC, and its use is not recommended according to current guidelines (choice A is incorrect). Performing additional dialysis at this point would have little effect on his bleeding diathesis (choice B is incorrect). Because hypofibrinogenemia can prolong both the PT and PTT, transfusing FFP is not the best next step (choice C is incorrect).

A 55-year-old man presents to the emergency department in police custody. He was witnessed to swallow a bag thought to contain methamphetamine, although he denies it. Vitals signs are T 98.1, BP 154/102, HR 90, and RR 17. What is the most appropriate disposition? A. Discharge after 4 hours if he remains stable without intervention and has a negative rectal exam B. Discharge after 8 hours if he remains stable without intervention and has a negative rectal exam and a negative abdominal x-ray C. Discharge after all reported baggies are recovered in the stool, rectal exam is negative, and an abdominal CT scan is negative D. Consider a dose of activated charcoal and discharge after 24 hours if he remains stable without evidence of toxicity and has a negative rectal exam

The correct answer is D. Rationale: This scenario meets the definition of body stuffing with risk for methamphetamine exposure from leakage of contents out of a package not intended for intestinal transit (body packing). Discharge should not occur before 24 hours (choice D is correct, and choices A and B are incorrect). Of note, waiting for all baggies is impractical as some may not be recovered (choice C is not the best answer although it might be a good answer for a body packer).

A 21-year-old man presents with a chief complaint of a spider bite 1 day ago that he says was a brown recluse. On examination, there is an erythematous, raised, warm lesion on the left forearm. His vitals are normal, he has no systemic symptoms, his urine is pale in color, and his pain is controlled with minimal analgesia. What is the most appropriate management? A. Administer recluse antivenom, half around the site, half intravenously B. Administer black widow antivenom, 1 vial C. Check CBC with diff, Type and cross, LDH, U/A and admit for observation D. Evaluate for methicillin-resistant Staphylococcus aureus (MRSA)

The correct answer is D. Rationale: While brown recluses can cause necrotic lesions in adults, their evolution is over several days and often the spider bite is unrecognized. Most often with this initial presentation the lesion turns out to be a skin MRSA infection (choice D is correct). There is no antivenom for brown recluse bites. The laboratories suggested might be appropriate for a child where the history was concerning for a recluse bite with systemic effects such as fever, weakness, and malaise.

A 35-year-old man presents 5 hours after an overdose. He admits to ingesting only atenolol and diphenhydramine (doses unknown). His vital signs are heart rate 88 beats/min, blood pressure 134/72 mm Hg, respiratory rate of 20/min, temperature of 36.8 °C, and oxygen saturation 99% on room air. Other than a depressed sensorium, his physical examination is unremarkable. What is the most appropriate next step in management? A. Check serum acetaminophen (APAP) level B. Give 2 mg physostigmine C. Give 5 mg of IV glucagon D. Monitor and consult psychiatry

The correct answer is A. Rationale: Even if a patient denies having ingested APAP, every patient presenting with an intentional overdose should have a serum acetaminophen concentration obtained, as roughly 1.5% of such patients will have a measurable level, and roughly 1 in 300 of those patients will need treatment with n- acetylcysteine (NAC) (choice A is correct). While this patient could potentially become symptomatic from the atenolol or diphenhydramine ingestions, the vital signs and physical examination are not consistent with either, so treatment with physostigmine or glucagon at this time is not indicated (choices B and C are incorrect). While every patient with intentional overdose should be seen by psychiatry, evaluation for and management of known and unknown ingestions is still in progress (choice D is not the immediate priority). Not listed as an answer choice, but there would also be no role for activated charcoal as it has been more than 2 hours since overdose.

A 24-year-old man presents to the emergency department with sharp chest pain. He admits to cocaine usage over the past 2 days. He has some diaphoresis and tachycardia but denies pain radiation, shortness of breath, fever, cough, or abdominal pain. On physical exam, vital signs include heart rate of 100 beats/min and blood pressure of 160/100 mm Hg. Pulmonary, cardiac, abdominal, and neurologic examinations are unremarkable. EKG shows early repolarization. Chest x-ray is unremarkable. Which of the following is the most appropriate medical therapy? A. Diazepam B. Nifedipine C. Metoprolol D. Ziprasidone

The correct answer is A. Rationale: First-line treatment for cocaine toxicity is benzodiazepines (choice A is correct). Nondihydropyridine calcium channel blockers such as diltiazem and verapamil have been shown to reduce hypertension reliably but not tachycardia. Dihydropyridine agents such as nifedipine should be avoided, as reflex tachycardia may occur (choice B is incorrect). β-blockers are theoretically contraindicated in cocaine toxicity due to unopposed alpha-receptor vasoconstriction (choice C is incorrect), though some studies do not show an increase in risk of myocardial infarction or all-cause mortality. Ziprasidone is a treatment for schizophrenia and manic/mixed bipolar disorder, not cocaine toxicity (choice D is not the best answer). In animal studies, it may limit drug dependence, but combination with cocaine has also reportedly caused dystonic reactions and other side effects.

From the following list of iron compounds, which is the least toxic? A. Carbonyl iron B. Ferrous fumarate C. Ferrous gluconate D. Ferrous sulfate

The correct answer is A. Rationale: Iron can damage gastrointestinal mucosa and impair oxidative phosphorylation and mitochondrial function. Toxicity can occur with all of the listed compositions, but carbonyl iron, a highly purified form of metallic iron that is uncharged and not a salt, is least toxic (choice A is correct).

A 32-week pregnant patient presents with hypertension and is admitted for preeclampsia workup and treatment. She admits to daily heroin use, with her last dose 12 hours ago. Her vital signs are HR 120 beats/min, BP 145/90 mm Hg, RR 20 breaths/min, T 37 °C. She exhibits mydriasis, lacrimation, rhinorrhea, diaphoresis, yawning, piloerection, anxiety, and restlessness. Which one of the following is the best treatment strategy for this patient? A, Methadone B. Morphine C. Ultrarapid opioid detoxification D. Wait 72 hours, then buprenorphine-naloxone (Suboxone)

The correct answer is A. Rationale: It is appropriate to initiate oral opioid substitution therapy to alleviate this patient's opioid withdrawal symptoms followed by daily maintenance therapy. Every attempt should be made to minimize significant opioid withdrawal manifestations here as it may adversely affect the developing fetus to include fetal distress and intrauterine death. Oral methadone maintenance treatment is more compatible with maternal and fetal well-being than continued heroin abuse and would likely decrease the risk of vertical human immunodeficiency virus transmission (choice A is correct). Methadone is often preferred to buprenorphine for maintenance, as induction with buprenorphine will usually precipitate mild withdrawal. Buprenorphine may be appropriate if her withdrawal was moderate at presentation, however it is not appropriate to leave withdrawal untreated for several days (choice D is incorrect). Ideally, buprenorphine induction is done when the patient is exhibiting early stages (eg, yawning and rhinorrhea) to minimize the risk of precipitating severe withdrawal. This is usually at least 6 hours after the last heroin use. The first buprenorphine dose is typically initiated after the onset of mild withdrawal, with a COWS score of at least 6, and preferably greater than 10. Dosing may be advanced by 4 mg/d on subsequent induction days as necessary to control withdrawal symptoms up to a total dose of 32 mg/d. A morphine taper would increase the chance of protracted withdrawal and is not recommended (choice B is incorrect). Ultrarapid opioid detoxification (administering high doses of an opioid antagonist (eg, naltrexone) and clonidine to individuals with opioid use disorder while under deep sedation or general anesthesia) has been suggested as a method of achieving detoxification within 24 to 48

A 23-year-old woman presents to the emergency department with altered mental status. She has a history of bipolar disorder and hypertension. She was found by her roommate after she was heard falling out of bed. An empty bottle of thirty 250 mg divalproex sodium (valproate) tablets was found nearby along with an empty beer can. On arrival, she has normal vital signs. Physical exam is notable for an obtunded-appearing female who makes incomprehensible sounds and withdraws all extremities from pain but does not open her eyes. GCS is 7. She has bilateral lower extremity clonus. A fingerstick glucose is 113 mg/dL. What is the most appropriate next step in management? A. Administer naloxone B. Consult nephrology for hemodialysis C. Intubate for airway protection D. Place nasogastric tube for activated charcoal followed by l-carnitine

The correct answer is A. Rationale: It is reasonable to try one or several doses of naloxone in a patient with suspected valproic acid (VA) overdose prior to intubation for airway protection (choice A is correct). There are several case reports of improvement in mental status with naloxone for patients with VA toxicity, and there is experimental evidence to support a biologically plausible mechanism for this effect. Intubation, administration of activated charcoal, and supplementation with l-carnitine would all be next steps if the patient does not have improvement with naloxone and/or develops respiratory compromise or distress. Hemodialysis may be indicated later in the patient's clinical course depending on the presence of shock, cerebral edema, or markedly elevated valproic acid level exceeding 1000 μg/mL.

A 57-year-old man is critically ill in severe congestive heart failure with an ejection fraction of 10%. After consultation with cardiac surgery, he undergoes placement of a left ventricular assist device (LVAD). Upon transfer back to the cardiac ICU, he is noted to have frank melena. He has no prior history of bleeding disorders. What is the most likely diagnosis? A. Acquired von Willebrand disease B. Disseminated intravascular coagulation (DIC) C. Gastric varices due to right heart failure D. Stress gastric ulcer

The correct answer is A. Rationale: LVAD placement is associated with acquired von Willebrand disease due to mechanical shearing of von Willebrand factor multimers by left ventricular assist device impellers (choice A is correct). Acute blood loss from stress ulcers, gastric varices, and DIC can all occur after cardiac surgery but would not be expected immediately.

An 18-year-old man presents after ingestion of a 7% hydrofluoric acid solution in a self-harm attempt. The patient reports ingesting several swallows of the product. He reports some epigastric discomfort but otherwise no symptoms. His vital signs are normal. He has no oral burns or symptoms of airway injury. What is the most appropriate next step in management? A. Begin empiric IV calcium administration B. Intubation and gastric lavage C. Oral administration of calcium carbonate D. Treatment only if hypocalcemia develops

The correct answer is A. Rationale: Patients with deliberate ingestion of hydrofluoric acid can develop rapid severe hypocalcemia and cardiovascular collapse. Early intravenous calcium administration is associated with survival (choice A is correct and D is not the best answer). Oral calcium carbonate and gastric lavage are not effective in this setting (choices B and C are incorrect).

Opioids with short half-lives may still have prolonged effects when A. Enzymes that metabolize drugs become saturated, leading to zero- order kinetics B. High serum concentrations of opioids can inhibit enzyme rates of metabolism, producing a synergistic effect C. Opioid receptor expression is upregulated, leading to increased opioid-receptor binding D. Supersaturation of opioid receptors causes the receptors to be perpetually occupied and continuously stimulated

The correct answer is A. Rationale: Saturation of cytochromes P450 (CYP) enzymes causes drug metabolism to convert from first-order (constant fraction metabolized per unit time) to zero-order (constant amount metabolized per unit of time) (choice A is correct). This, along with prolonged absorption that may occur due to opioid- induced ileus, can lead to the greatly prolonged toxicokinetics seen during overdose. The rate of opioid degradation by metabolic enzymes is largely independent of opioid substrate concentration (choice B is incorrect). Opioid receptor expression or binding is not a major determinant of their rates of degradation (choices C and D are incorrect).

A 68-year-old man with a history of paroxysmal atrial fibrillation presents to the emergency department because of fatigue, dizziness, and generalized weakness over the past week. His medications include losartan, digoxin, and rivaroxaban. Examination of the patient is remarkable for a regular heart rate of 38 beats per minute. Electrocardiogram confirms sinus bradycardia. Which of the following added findings would be an indication for immediate treatment with digoxin-specific antibody fragments? A. Blood pressure 72/44 B. Patient report of green-yellow visual halos C. Serum digoxin concentration 2.3 ng/mL D. Serum potassium concentration 3.0 mEq/L

The correct answer is A. Rationale: Symptomatic bradycardia, ventricular dysrhythmias, and hemodynamic instability are among the indications for immediate, empiric digoxin-specific antibody fragments (Fab) therapy (choice A is correct). Patients with chronic digoxin poisoning as suggested by the visual halos (choice B), and elevated concentrations (choice C), may be treated with supportive care and monitored by cardiac telemetry when hemodynamically stable, though, even then, Fab should be considered if multiple medical comorbidities are present. Serum potassium concentration >5.0 mEq/L is an indication for digoxin-specific antibody fragments therapy, not hypokalemia (choice D is incorrect).

What mechanism is most likely responsible for the clinical manifestations of theophylline toxicity? A. Adenosine receptor antagonism and catecholamine release B. Phosphodiesterase inhibition and increased cyclic adenosine monophosphate C. Potassium channel antagonism and sodium-potassium adenosine triphosphatase (Na+/K+-ATPase) inhibition D. Sodium and calcium channel antagonism

The correct answer is A. Rationale: The physiological changes seen with therapeutic theophylline dosing, including tachycardia, diuresis, bronchodilation, and CNS excitation, were thought to result from theophylline's inhibition of phosphodiesterase, the intracellular enzyme that inactivates cAMP. Such enzyme inhibition would lead to elevated intracellular cAMP concentrations. However, this theory has been brought into question. In vitro data suggest phosphodiesterase inhibition does not occur at therapeutic serum concentrations. Whether the increased theophylline concentrations seen in the intoxicated patient are sufficient to inhibit phosphodiesterase activity is unclear. Investigation has also been directed at the role of adenosine receptor antagonism. Adenosine is a nucleoside that promotes smooth muscle constriction, slows cardiac conduction, and acts as an endogenous anticonvulsant. The chemical structures of theophylline and adenosine are similar and so theophylline may be a simple competitive antagonist, but stimulation of plasma catecholamine release is also likely involved. With therapeutic theophylline dosing, plasma catecholamine activity typically increases four- to sixfold. In theophylline intoxication, plasma catecholamine activity may rise 30-fold. Increased plasma catecholamines provide a ready explanation for the many effects of theophylline seen after therapeutic dosing and, potentially, many of the effects of theophylline intoxication. Therefore, the combined effects of adenosine receptor antagonism and catecholamine release, rather than PDE inhibition and cAMP increases, are probably responsible for most clinical features of theophylline toxicity (choice A is correct).

A 50-year-old man presents to the emergency department with palpitations for the past 2 hours. The patient denies medical problems, though admits to having not been to a physician in years. Palpitations have occurred twice before but terminated on their own. He denies fever, chest pain, or shortness of breath. Cardiac exam reveals an irregularly irregular rhythm. Electrocardiogram shows likely preexcitation with atrial fibrillation. Labs demonstrate a glomerular filtration rate (GFR) of 30 but are otherwise reassuring. The patient inadvertently receives a dose of procainamide that was not adjusted for renal function. What is the most likely adverse effect of this medication error? A. Hypotension B. Lupus-like syndrome C. QRS widening D. Seizure

The correct answer is A. Rationale: The primary effect of acute procainamide toxicity is hypotension (choice A is correct). A syndrome resembling systemic lupus erythematosus can be seen in patients receiving long-term oral therapy. This occurs in about 40% of patients, usually resolves after drug withdrawal, and would not be expected to happen acutely (choice B is incorrect). While QRS widening certainly can occur, it is not as likely as hypotension (choice C is not the best answer). Procainamide is not known to cause seizures or have other neurological adverse effects (choice D is incorrect).

A previously healthy 20-year-old man is brought to the emergency department (ED) after collapsing while dancing at a night club. At arrival he was unconscious with a Glasgow Coma Score 3, heart rate 56 beats per minute, blood pressure 110/56 mm Hg, and a temperature 36 °C. Occasional myoclonic jerks were noted. Pupils were small but reactive. There was no response to a total of 800 μg of IV naloxone and he was endotracheally intubated and ventilated for airway protection. Blood sugar was normal and head CT scan revealed no acute abnormality. Over the next few hours, he required escalating doses of propofol for sedation, culminating in him rapidly self-extubating6 hours after initial presentation. He was guarded regarding the events leading to his presentation, and promptly discharged himself from the hospital against medical advice. Which of the following drug ingestions was most likely responsible A. γ-hydroxybut

The correct answer is A. Rationale: This is a classic clinical picture of γ-hydroxybutyrate overdose (GHB), with rapid loss of consciousness, relative bradycardia, miosis, and myoclonic jerks, followed by a rapid recovery in conscious state (choice A is correct). The clinical history and lack of response to naloxone make heroin overdose unlikely (choice B is incorrect). The speed of recovery is not characteristic of phenobarbital poisoning (choice C is incorrect). The depth of coma makes zopiclone less likely (choice D is incorrect).

A 24-year-old woman presents 45 minutes after an intentional ingestion of approximately 25 g of acetaminophen (APAP). She denies any other ingestions. Her vital signs are heart rate 80 beats/min, blood pressure 128/62 mm Hg, respiratory rate of 18/min, temperature of 37 °C, and oxygen saturation 99% on room air. Her physical examination is normal except for tearfulness. What is the most appropriate next step in management? A. Give activated charcoal B. Give intravenous N-acetylcysteine (NAC) therapy C Measure serum APAP level D. Obtain ECG

The correct answer is A. Rationale: This is a significant ingestion, and activated charcoal is indicated as it has been less than 2 hours since ingestion (choice A is correct). There is no indication or benefit to immediate treatment with oral or intravenous NAC at less than 1 hour post ingestion (choice B is incorrect). While an APAP blood concentration is needed, obtaining one within an hour is neither necessary nor helpful (choice C is incorrect). An ECG is indicated after all intentional ingestions, but at the time of presentation oral activated charcoal is of greater benefit than an ECG (choice D is not the best answer). Not listed here as a choice, but in the absence of severe liver injury or other prognostic factors that predict need for liver transplantation, there is also no indication to transfer to a transplantation center.

A 50-year-old man is admitted to the hospital with pneumonia and started on appropriate antibiotics. On day 2 of his admission, he is diaphoretic, tremulous, anxious, and complains of headaches. His vital signs are HR 130 beats/min, BP 150/90 mm Hg, RR 18 breaths/min, T 37 °C, pulse oximetry 98% (room air). He has intermittent periods of confusion. Complete blood count is normal, and liver enzymes are mildly elevated but serum ammonia level is normal. His wife reports he has heavy alcohol intake every evening before bed. Which one of the following is the most appropriate treatment for this patient? A. Benzodiazepine B. Phenothiazine C. Propofol D. Thiamine

The correct answer is A. Rationale: This patient clinical course—including the newly revealed history —is consistent with acute alcohol withdrawal. While additional work-up to rule out other conditions is appropriate, a Clinical Institute Withdrawal Assessment of Alcohol Scale, Revised (CIWA-Ar) score should be obtained and treatment initiated to prevent further decompensation. First line treatment is benzodiazepines using a symptom-triggered approach based on that CIWA-Ar score (choice A is correct). An antipsychotic will not prevent further decline of his overall withdrawal syndrome or the development of seizures (choice B is incorrect). Propofol may be appropriate as an adjunctive agent for refractory withdrawal if there is an incomplete response to benzodiazepines, but it is not first line therapy (choice C is incorrect). Thiamine is appropriate for prophylaxis and treatment of Wernicke's encephalopathy but is an inappropriate single agent treatment for acute alcohol withdrawal (choice D is incorrect). If a patient with severe alcohol withdrawal is unresponsive to large doses of a benzodiazepine, additional therapies to include barbiturates may be warranted.

A 76-year-old man presents to the emergency department after a witnessed seizure lasting one minute which self-terminated shortly prior to arrival. He has a history of congestive heart failure, hypertension, and 2 diabetes complicated by neuropathy. He is prescribed furosemide, lisinopril, and metformin by his primary care physician and was also recently started on gabapentin 300 mg TID after visiting an urgent care clinic complaining of pain in his feet. His vital signs are normal, and his exam is remarkable for dry mucous membranes, bilateral upper extremity clonus, and a GCS of 13 (E3V4M6). He is asking about going home. Workup in the emergency department is notable for a creatinine of 2.76 mg/dL (most recent value 3 months ago was 1.4 mg/dL), BUN 76 mg/dL, sodium 133 mEq/L, potassium 5.2 mEq/L, glucose 146 mg/dL, WBC 13,000/ μL, and Hgb 10.8 g/dL. Estimated glomerular filtration rate is 26 mL/min. A CT scan of t

The correct answer is A. Rationale: This patient has an acute kidney injury (AKI) and seizure likely secondary to gabapentin toxicity. Treating his volume depletion and continuing to monitor are the most appropriate actions (choice A is correct). Starting an antiepileptic drug such as levetiracetam would not treat the underlying cause of the seizure (choice C is incorrect). Discharge would be inappropriate given the degree of acute kidney injury and it would also not address the underlying cause of the seizure (choice D is incorrect). Consulting neurology for EEG may be of benefit if the patient continues to have seizures after renal function improves (choice B is not the best answer).

A 64-year-old woman is brought to the ICU after sustaining an intracranial bleed in a motor vehicle collision. She is intubated and sedated, and you are unable to obtain a history. Her pharmacy reports that she is on dabigatran for atrial fibrillation. It is unclear when she took her last dose. Relevant labs include a hemoglobin of 6 g/dL (normal 12-16 g/dL), platelets 110,000/mm3 (normal 150-400,000/mm3), PT 11 s (normal 9.5-11.7 s) with INR 1.1, and PTT 28 s (normal 23-32 s). What is the most appropriate next step in management? A. Administer fresh frozen plasma (FFP) B. Administer idarucizumab C. Administer prothrombin complex concentrate (PCC) D. No specific management given her PT/PTT

The correct answer is B. Rationale: Idarucizumab, is a F(ab)2 (antibody fragment) against dabigatran, which binds and clears active drug from the circulation and is the definitive treatment for dabigatran-associated bleeding (choice B is correct). Reversal of dabigatran effects is indicted with CNS bleeding, and standard tests are often normal in this situation (choice D is incorrect). FFP may be indicated as part of her trauma resuscitation but is not effective for dabigatran-associated hemorrhage (choice A is incorrect). PCC may be helpful, but it is not definitive treatment (choice C is incorrect).

A 64-year-old man presents to the emergency department with complaints of nausea and mild abdominal pain for 2 days. One week prior, he underwent a total hip arthroplasty. His medications include atorvastatin, lisinopril, apixaban, and acetaminophen-oxycodone. His vital signs are heart rate 110 beats/min, blood pressure 140/75 mm Hg, respiratory rate 20/min, temperature 37 °C, and oxygen saturation 98% on room air. His physical examination reveals right upper quadrant tenderness to palpation. There is no scleral icterus. Laboratory studies include an AST of 6500 IU/L, ALT 5500 IU/L, total bilirubin of 2.2 mg/dL, INR of 1.1, lipase of 55 U/L, creatinine of 1.4 mg/dL, with an acetaminophen concentration of 20 μg/mL. What is the most appropriate next step in management? A. Administer N-acetylcysteine (NAC) B. Consult hepatology for ERCP C. Fomepizole infusion D. Transfuse plasma

The correct answer is A. Rationale: This patient likely has subacute/chronic acetaminophen toxicity. Further history from the patient should be obtained to determine exactly how much acetaminophen he has consumed (from all sources) but NAC infusion should be started now as he has measurable acetaminophen in his blood combined with evidence of liver injury (choice A is correct). There is no evidence of choledocholithiasis, so an ERCP is not indicated (choice B is incorrect). While fomepizole might have a role in an acute and massive acetaminophen overdose, it is not indicated for this case (choice C is incorrect). There is also no indication for plasma transfusion at this time (choice D is correct).

A 57-year-old woman presents to the emergency department after an intentional overdose. Her past medical history includes trigeminal neuralgia for which she was prescribed carbamazepine 600 mg/d. She is brought to the emergency department by her husband approximately 30 minutes after he witnessed her take sixty 200 mg carbamazepine ER tablets. On presentation, vital signs include a heart rate 67 beats/min, blood pressure 125/78 mm Hg, RR 15/min, T 37 °C, and SpO2 98% on room air. Weight is 60 kg. She is tearful but cooperative, and her GCS is 15. Exam is otherwise notable only for mild mydriasis. Initial laboratory values include WBC 12,300/μL, Hb 12.8 g/dL, platelets 256,000/μL, sodium 138 mEq/L, potassium 4.6 mEq/L, HCO3 23 mEq/L, Chloride 99 mEq/L, BUN 23 mg/dL, creatinine 1.05 mg/dL, glucose78 mg/dL, aspartate aminotransferase (AST) 17 U/L, alanine transaminase (ALT) 24 U/L, acetaminophen <10 μg/mL, salicylat

The correct answer is A. Rationale: This patient presents after a massive ingestion of carbamazepine (approximately 200 mg/kg) and is expected to have major toxic sequelae. She should be admitted to a level of care capable of carrying out rapid sequence intubation when this occurs (choice A is correct). Carbamazepine absorption from the GI tract is erratic and peak levels have been reported to occur between 8 and 60 hours after ingestion. Given the likelihood of delayed neurological deterioration, the risk of administering a dose of activated charcoal prior to securing her airway outweighs the benefit (choice B is incorrect). Sodium bicarbonate therapy would not be indicated at this point given the lack of any apparent cardiotoxic sequelae from the overdose (choice C is incorrect). Similarly, there are no indications for extracorporeal enhanced elimination therapies this early in her clinical course (choice D is incorrect) but she should be admitted to a unit capable of carrying out dialysis should she develop deep coma, refractory seizures, or life-threatening dysrhythmias.

A 26-year-old woman, her 29-year-old husband, and their 22-month-old daughter present to the emergency department for evaluation. All have felt ill for the past 2 days with fever, cough, vomiting, and diarrhea. All were previously healthy. They recently moved into a small room heated by a radiator above the maternal grandfather's garage, as they recently lost their own housing. In the room were several open and unmarked containers situated on top of the radiator. The parents moved these to a high shelf out of the child's reach and are confident she did not ingest anything. On physical exam, all three are febrile, tachycardic, and tachypneic, with coarse breath sounds on lung auscultation. Which one of the following investigations is most likely to establish the correct diagnosis? A. 24-hour urine mercury excretion B. 24-hour urine total arsenic concentrations C. Heavy metal content in hair and nail segments D. Wh

The correct answer is A. Rationale: This presentation is consistent with exposure to mercury vapor in the small, enclosed space over a few days. Fever, cough, vomiting, and diarrhea are nonspecific signs and symptoms and could be due to an infectious etiology, but all answer choices relate to poisoning. Arsenic poisoning would have severe gastrointestinal symptoms as the initial clinical manifestation and would not be expected to cause fever (choice B is incorrect). Testing for heavy metals in hair and nail segments is sometimes useful in documenting remote exposures but is inappropriate for investigating the acute exposures in this clinical scenario (choice C is incorrect). Lead poisoning can present with a variety of nonspecific symptoms that may overlap some with elemental mercury vapor poisoning. However, lead poisoning would not be expected to cause fever or cough (choice D is incorrect).

A 2-year old healthy child on no medications presents to Arizona emergency department. He has been inconsolable, agitated, and hysterical for the last hour. Initial vital signs are positive for tachycardia and tachypnea, but no fever. Exam demonstrates clear lungs, normal eye exam, and all four extremities are normal with no skin lesions. The tap test is impossible to conduct due to the child's nature. What is the most appropriate next step in management? A. Benzodiazepine B. Centruroides immune F(ab')2 antivenom C. Crotalid Antivenom FabAV—10 vials D. Cr otalid Antivenom F(ab')2AV—10 vials

The correct answer is A. Rationale: While some of the symptoms this child is having are consistent with Centruroides (bark scorpion) envenomation, with no increase in secretions and no ocular findings (no opsoclonus) that diagnosis should be questioned. The tap test or tapping in the area of a suspected scorpion sting to see if pain gets worse, will be impossible to conduct with this child. Initial treatment for a Centruroides sting would be benzodiazepines (choice A is correct) with antivenom reserved for systemic symptoms. Acute amphetamine toxicity can appear similar and would initially be treated with benzodiazepines and potentially antipsychotics depending on the response.

Which of the following is true about methamphetamine production in the United States today? A. Production has decreased due to imports B. Production has increased due to "shake and bake" technique C. Production has shifted to MDMA D. Production with liquid ammonia has increased risks to bystanders

The correct answer is A.Rationale: Methamphetamine laboratories are less common in the United States as importation from foreign, large-scale production facilities has become more common (choice A is correct, and choice B is incorrect). Methamphetamine is the most abundant product as MDMA is a more difficult synthesis requiring very different starting materials. (choice C is incorrect). The more dangerous Birch method of methamphetamine production uses anhydrous (a very cold, caustic liquid) rather than liquid ammonia (an aqueous solution of ammonia; (choice D is incorrect).

What is the most common clinically significant adulterant found in methamphetamines today? A. Benzocaine A 55-year-old man with suspected methamphetamine ingestion suddenly B. Fentanyl C. Lead D. Talc

The correct answer is B. Rationale: The most common, dangerous adulterant of methamphetamines is fentanyl (choice B is correct, and choices A and C are incorrect). Though, of note, many contaminants are possible to include beyond those listed. In fact, talc is commonly used as a filler to expand the weight and volume of methamphetamine preparations and is usually clinically insignificant in the short term. (choice D is incorrect although chronic intravenous use can result in pulmonary deposition).

A 28-year-old woman was brought to the emergency department by coworkers at a farm because of confusion. Her vitals are HR 46 beats/min, BP 74/48 mm Hg, RR 28/min, T 37.1 °C, and pulse oximetry is 91% on room air. On examination, she has miotic pupils, is diaphoretic, has copious frothy secretions from her mouth, and is vomiting. Point-of-care glucose is 108 mg/dL. Which one of the following is the most appropriate treatment after managing this patient's airway? A. Atropine B. Crystalloid fluid bolus C. Glucagon D. Norepinephrine E. Pralidoxime

The correct answer is A.Rationale: This patient presented with acute cholinergic crisis most likely as a result of exposure to an organophosphorus agent used as a pesticide on a farm. Acute cholinergic crisis manifests as excessive acetylcholine accumulation at the muscarinic receptors. Muscarinic and/or nicotinic symptoms can manifest. This patient is exhibiting signs of excess muscarinic stimulation which can be remembered by the mnemonic DUMBBBELS (diarrhea, urination, miosis, bronchospasm, bronchorrhea, bradycardia, emesis, lacrimation, salivation). Atropine muscarinic receptor antagonism is the mainstay antidotal therapy (choice A is correct). Goals of early therapy are directed at reversing cholinergic features and improve respiratory and cardiac dysfunction. The initial dose of intravenous atropine is 1 to 3 mg, which is doubled every 5 minutes with goal of reversing bronchospasm and bronchorrhea and raising heart rate to greater than 80. Once this is achieved, an atropine infusion may be required. A crystalloid fluid bolus is usually required to replace excessive fluid losses but will not treat acute cholinergic crisis (choice B is incorrect). Glucagon for hypoglycemia or suspected beta-blocker/calcium channel blocker toxicity; it has no role for acute cholinergic crisis (choice C is incorrect). Norepinephrine can be given for additional support of blood pressure and heart rate but does not treat the underlying muscarinic excess responsible for the acute cholinergic crisis (choice D is incorrect). Pralidoxime is an oxime that is effective at the nicotinic receptors to improve muscle strength (eg, respiratory muscles) where atropine is ineffective (choice E is not first line treatment). The antidotal effect of oximes and atropine is synergistic because oximes reactivate AChE while atropine inhibits acetylcholine

An 18-month-old presents after a small ingestion of household bleach. The child spit it out and was gagging initially but now is improving. Vital signs are within normal limits and there is some mild oral redness. The exam is otherwise normal and the child is taking fluids. Which of the following is the most appropriate next step? A. CT scan of neck, chest, and abdomen with contrast B. Discharge home with return precautions C. Endoscopy now for risk stratification D. Inpatient monitoring with delayed endoscopy in 24 hours

The correct answer is B. Rationale: Accidental household bleach ingestions are very low risk and generally do well (choice B is correct). CT scanning and endoscopy are not indicated (choices A, C, and D are incorrect).

A 65-year-old woman with a history of myelodysplastic syndrome (MDS) is admitted to the hospital with fever, chills, and shortness of breath. She has received multiple blood products (RBCs and platelets) over the past 2 months. She is found to have possible sepsis and disseminated intravascular coagulation (DIC) due to pneumonia. She no longer appears to be responding to platelet transfusions, as evidenced by no or only minimal increase in platelet counts obtained 30 minutes after transfusion. What is the most likely reason for her being refractory to platelet transfusions? A. Disseminated intravascular coagulation (DIC) B. HLA alloimmunization C. Neutropenic fever D. Sepsis

The correct answer is B. Rationale: All answers listed are associated with less-than-optimal increases in platelet counts post transfusion. However, when recipients fail to have increments in platelet counts obtained within 1 hour of transfusion, the most likely cause of refractoriness is immune-mediated destruction (choice B is correct). Nonimmune causes of platelet refractories usually have an initial increase in platelet count followed by decreasing counts over the subsequent 24 hours. The antibodies most implicated in immune-mediated platelet refractoriness are directed against class I HLA antigens.

A 55-year-old man with suspected methamphetamine ingestion suddenly becomes altered, tremulous, diaphoretic, and agitated about 2 hours after arrival to the emergency department. He is intubated but remains agitated. His temperature is 99.1 and no seizure activity is noted. What is the most appropriate medical management? A. Antipsychotics B., Benzodiazepines C. Centrally acting α-antagonists D. Opiates

The correct answer is B. Rationale: Benzodiazepines are the agent of choice for control of agitation after acute xenobiotic (toxin) exposure (choice B is correct). Antipsychotics, opioids, and barbiturates may be effective in controlling agitation but are associated with higher rates of seizures and respiratory depression in this setting (choices A and D are not the best answers). Additionally, agents that manipulate mediators of the sympathetic nervous system, such as alpha antagonists, risk paradoxical effects and should be avoided (choice C is incorrect).

Which of the following test results is most likely associated with severe iron toxicity? A. Iron tablets visible on x-ray, 4 hours after ingestion B. Presence of metabolic acidosis C. Serum iron concentration greater than serum TIBC (total iron binding concentration) D. Serum iron concentration of 300 μg/dL, 4 hours after ingestion

The correct answer is B. Rationale: Blood gas or serum bicarbonate determinations should be done early in evaluation because metabolic acidosis is an early, objective indicator of systemic toxicity (choice B is correct). Peak iron serum level occurs 4 to 6 hours after ingestion. Sustained release/enteric coated tablets may have peak levels at 8 hours. Mild to moderate toxicity begins at levels of 500 (choices C and D are incorrect). There is no correlation between radiopacities on x-ray and severity of poisoning (choice A is incorrect).

A 65-year-old man is admitted to an intensive care unit following an intentional β-blocker overdose. He remains hypotensive despite intravenous fluid boluses and an epinephrine drip. An echocardiogram shows significantly depressed myocardial contractility. Which one of the following should be considered next as adjunctive therapy to epinephrine for this patient? A. Enoximone B. Hyerinsulinemic euglycemia C. Lipid emulsion D. Vasopressin

The correct answer is B. Rationale: Hyperinsulinemic euglycemia therapy is the most likely to improve inotropy (choice B is correct). The clinical experience with phosphodiesterase inhibitors such as enoximone is limited to a few case reports (choice A is incorrect). While lipid emulsion therapy may create a lipid sink for lipophilic drugs, augment cardiac energy substrates, and have some inotropic affect, it should be reserved for refractory cases (choice C is incorrect). Vasopressin and other vasopressors will cause vasoconstriction, transiently increasing blood pressure, which can be detrimental in the face of poor inotropy (choice D is incorrect).

A 2-year-old child presents to the emergency department at 6pm after being bitten by a copperhead. The bite is to the calf, and there are two oozing puncture wounds about 1.5 cm apart with some mild local erythema. The calf is soft, and there is no swelling. Vitals are normal, and physical exam is otherwise unremarkable. Over the first few hours of observation the patient has minimal pain. What is the most appropriate management? A. Administer IV Crotalid Antivenom B. Admit for observation overnight C. Discharge now with follow-up in 3 days D. Observe for a total of 6 hours, if no progression, then follow up in 3 days

The correct answer is B. Rationale: Lower extremity bites from copperheads especially to ankle or calf can have significant delay in progression of swelling, so admission is warranted (choice B is correct). Children may also have greater extremity injury from a copperhead bite relative to adults as their limbs are smaller and the venom load is larger per unit weight. With a full 8 hours observation this patient could be discharged if he has an excellent support network, negative/normal laboratories at 6 hours, and a follow-up 8 hours after discharge (usually by phone) but waiting 3 days is too long (choices C and D are incorrect). For this patient who has what looks to be a "dry" bite (a bite without venom administration), antivenom should not be used at this point (choice A is incorrect).

Which of the following statements about theophylline is true? A. It has the same physiological actions as adenosine B. Its clearance is decreased by CYP1A2 and CYP3A4 inhibitors C. It is metabolized to caffeine D. Seizures from its toxicity should be treated with phenytoin

The correct answer is B. Rationale: Many xenobiotics, chemicals, and medical conditions affect the steady-state serum concentration and elimination half-life of theophylline. The xenobiotics that inhibit theophylline clearance are those that inhibit CYP1A2 and CYP3A4 (choice B is correct). The chemical structures of theophylline and adenosine are similar but theophylline acts as an adenosine receptor antagonist and the two drugs thus have generally opposing physiological actions (choice A is incorrect). Theophylline appears to be a simple adenosine competitive antagonist at bronchial and vascular smooth muscle, cardiac, and CNS sites although adenosine antagonism alone does not provide a complete explanation for theophylline's pharmacologic effects. In all probability, the combined effects of adenosine receptor antagonism and catecholamine release are responsible for most clinical features of theophylline intoxication. The pharmacokinetic profile of caffeine resembles theophylline, with an important exception: whereas metabolism of theophylline (1,3-dimethylxanthine) produces inactive metabolites, caffeine (1,3,7-trimethylxanthine) undergoes 7-demethylation to form theophylline (choice C is incorrect). Seizures should be treated aggressively. High-dose benzodiazepine may be necessary for seizure termination. Phenytoin may be ineffective for theophylline-induced seizures, and in animal studies, it appears to even worsen them (choice D is incorrect).

What is the most common toxicity seen after petroleum distillate ingestion? A. Altered mental status and seizures B. Aspiration and chemical pneumonitis C. Cardiac dysrhythmias D. Gastrointestinal bleeding

The correct answer is B. Rationale: Petroleum distillates are hydrocarbon solvents produced from crude oil. These solvents include mineral spirits, kerosene, and white spirits. They are good for removal of heavy oil and grease, but these properties also mean they have a tendency to get in the airway when ingested (choice B is correct). Other choices listed are less common, though lethargy and seizures can be seen with substantial absorption of halogenated hydrocarbons, and cardiac dysrhythmias can be fatal (choices A, C, and D are not the best answers).

A 24-year-old presents after ingestion of toilet bowl cleaner in a self-harm attempt. The patient has oral burning and drooling. Vital signs include a heart rate of 120 beats/min, BP of 140/82 mm Hg, respiratory rate of 24 breaths/min, and temperature of 37 °C. The patient has oral redness and difficulty swallowing but an otherwise unremarkable exam. After any airway management and resuscitation, what is the most appropriate next step? A. Endoscopy B. Contrast CT scan of neck, chest, and abdomen C. Monitoring including delayed endoscopy D. Surgical exploration

The correct answer is B. Rationale: Recent studies have shown CT-based evaluation is less likely to result in negative surgical exploration and have similar outcomes (choice B is the best next step). Endoscopy and surgical exploration may be required after initial evaluation (choices A, C, and D are incorrect).

Which of the following metabolic profiles would be consistent with calcium channel blocker (CCB) toxicity? A. Hyperglycemia and ketoacidosis B. Hyperglycemia and lactic acidosis C. Hypoglycemia and ketoacidosis D. Hypoglycemia and lactic acidosis

The correct answer is B. Rationale: Severe calcium channel blocker poisoning has a high mortality rate, with a third of cardiovascular drug overdose deaths being due to them. They cause cardiovascular depression, which can be refractory to standard resuscitation methods. Metabolic changes also occur with poisoning and include a state of acidosis and hyperglycemia. This may appear like diabetic ketoacidosis; however, lactate, as opposed to ketones, is the most prominent derangement (choice B is correct).

A 19-year-old woman with history of depression is found unresponsive on the floor of her grandparent's bathroom with an empty bottle of pills. She is breathing spontaneously and is transported to the nearest hospital. After the patient is put on cardiac telemetry she suddenly goes into pulseless bidirectional ventricular tachycardia. Cardiopulmonary resuscitation is initiated. Emergency medical personnel do not know what the empty pill bottle contained but you suspect digoxin. What is the digoxin-specific antibody fragments dose to administer to the patient? A, 1 to 2 vials B. 5 to 10 vials C. 20 to 30 vials D. Calculate once serum digoxin level is known

The correct answer is B. Rationale: The empiric dose of digoxin-specific antibody fragments to administer to a hemodynamically unstable patient following an acute overdose is 5 to 10 vials (choice B is correct). This dose should be given as a bolus in the setting of imminent or active cardiac arrest. One or two vials may be appropriate in the setting of chronic digoxin poisoning without impending cardiac arrest, but not in this emergent acute case (choice A is incorrect). Twenty to 30 vials is more than required as initial dose for most acute overdoses, though a patient may end up needing 20 or more vials throughout their hospitalization (choice C is incorrect). During cardiac arrest or imminent arrest, patients should receive empirical treatment. Physicians should not wait to calculate total body burden of digoxin (choice D is incorrect).

A 29-year-old man cleaning a chicken coop was bitten through gloves by a snake described as having a "diamond-shaped" pattern. There are timber rattlesnakes and copperheads in the area. He has a single puncture wound to his index finger. Symptoms include paresthesia, bruising and swelling to the finger, and, 30 minutes later, he has swelling up to but not past his wrist. His vital signs are normal, and he has no systemic symptoms. What is the most appropriate next step in management? A. Crotalid antivenom FabAV—10 vials B. Crotalid antivenom F(ab')2AV—6 vials C. Elevate arm, check PT/INR, platelets and fibrinogen 6 hours post bite, and reassess D. North American Coral Snake Antivenom—3 vials

The correct answer is B. Rationale: This appears to be a pit viper envenomation and given the amount of swelling over such a short period of time the patient should be treated. Either the FabAV or F(ab')2AV are acceptable, but the initial dosing for the FabAV is typically 4 to 8 vials for a probable rattlesnake bite, not 10 vials (so choice B is correct). Coral snakes generally bite and latch on, do not often (choice D is incorrect) cause one or two puncture wounds, and do not cause this much local swelling. While elevating the hand and obtaining laboratory testing is important, this patient has enough clinical reason now to treat with antivenom without further delay (choice C is incorrect).

A 53-year-old man with hypertension is admitted to the intensive care unit for altered mental status. His wife reports that for several months her husband has been complaining of headaches, difficulty concentrating, myalgias and difficulty sleeping, and has overall seemed withdrawn. He has had chronic constipation for years that has been manageable with fiber supplementation, but more recently has become worse and accompanied by intermittent, severe abdominal cramping. Today, he became acutely confused and seemed unsteady on his feet, so his wife brought him in for evaluation. On physical examination, his vital signs are remarkable for a blood pressure of 148/92 mmHg. He is alert but disoriented, is tremulous with a wide-based gait, and has a benign abdominal examination. Laboratory investigations reveal a creatinine of 2.3 mg/dL and a hemoglobin of 9.1 g/dL. Which one of the following would explain these findings? A

The correct answer is B. Rationale: This case describes lead toxicity. Of all the answer choices, only working in a battery manufacturing plant would put this patient at risk for significant lead exposure (choice B is correct). A diet heavy in large, predatory fish would exposure the patient to a variety of heavy metals including arsenic and mercury. Though lead does bioaccumulate in large predatory fish, it is primarily concentrated outside of the muscle, so heavy consumption of fish meat is not a significant risk factor for lead toxicity (choice A is incorrect). Silver fillings used as dental amalgams contain mercury, not lead (choice C is incorrect). While lead may be present in some traditional remedies, cinnabar contains mercury (choice D is incorrect).

A 48-year-old woman is brought in after being found unconscious in a house fire. She was intubated in the emergency department and transferred to the ICU for further management. She received 2L IV Ringer lactate and is now on a norepinephrine infusion. Her Glasgow Coma Scale (GCS) score has remained at 3 since arrival. Her vitals are temperature 37.2 C, HR 120/min, BP 70/50 mm Hg, respiratory rate 18 set by ventilator, and oxygen saturation 95% on 100% FiO2. Blood glucose is 8.5 mmol/L. Initial laboratory testing reveals pH 6.98, pCO2 34 mm Hg, bicarbonate 6 mmol/L, and lactate 17 mmol/L. What is the most appropriate next step in management? A. Dantrolene B. Hydroxocobalamin C. Hyperbaric oxygenation D. Methylene Blue

The correct answer is B. Rationale: This clinical presentation is highly suggestive of cyanide toxicity. Patients with hyperlactatemia after a fire should be assumed to have inhaled cyanide. Obtundation, hypotension, and tachycardia all support the diagnosis. Treatment of cyanide toxicity is prompt administration of hydroxocobalamin (choice B is correct). Dantrolene would be used in patients with significant hyperthermia and muscle rigidity secondary to malignant hyperthermia (choice A is incorrect). Carbon monoxide toxicity is also a concern after a fire, and a carboxyhemoglobin level should certainly be obtained. However, given the elevated lactate and hemodynamic instability, management of suspected cyanide takes precedence over transfer to a hyperbaric chamber for carbon monoxide toxicity (choice C is incorrect). Methylene blue is therapy for methemoglobinemia (choice D is incorrect).

A 38-year-old previously healthy man is admitted to the ICU for evaluation of new onset severe anemia. He is jaundiced and has scleral icterus. A hemolytic anemia is suspected. Which of following laboratory findings are most consistent with this diagnosis? A. Elevated haptoglobin B. Elevated reticulocyte count C. Low lactate dehydrogenase (LDH) D. Low total and indirect bilirubin

The correct answer is B. Rationale: This patient has a new onset hemolytic anemia. As he was previously healthy, the expected response would be to compensate for the anemia with increased bone marrow production of red blood cells (ie, reticulocytes) as reflected by an increased reticulocyte count (choice B is correct). Typically, hemolytic anemias are normocytic or macrocytic, not microcytic. The haptoglobin is typically decreased (choice A is incorrect), LDH is increased (choice C is incorrect), and indirect bilirubin is increased (choice D is incorrect).

A 28-year-old man presents after overdosing on aripiprazole. Time since ingestion is unknown. His heart rate is 120 beats/min, blood pressure 82/58 mm Hg, respiratory rate 18 breaths/min, temperature 37 °C, and oxygen saturation 98% on room air. His skin is warm and dry, with some axillary sweat present. His mental status is slightly depressed but he does arouse with stimulation. Pupils are 2 to 3 mm bilaterally and reactive. Reflexes are 2+ symmetrically. An ECG demonstrates PR of 180 ms, QRS of 116 ms, and QTc of 420 ms. What is the most appropriate next step in management? A. Activated charcoal via nasogastric tube B. Intravenous crystalloid fluid bolus C. Naloxone 0.4 mg IV D. Physostigmine 2 mg IV

The correct answer is B. Rationale: This patient has hypotension and mild tachycardia, common manifestations of large antipsychotic overdoses, which should be treated with volume repletion (choice B is correct). Given the unknown time from ingestion and the presence of clinical effects, there is no indication for activated charcoal now via any route (choice A is incorrect). This patient's presentation is not consistent with opioid effects, so there is no role for naloxone (choice C is incorrect). While this patient may have some degree of anticholinergic toxidrome, the presence of axillary sweat and nonmydriatic pupils suggest he does not need physostigmine at this time (choice D is incorrect).

An 80-year-old woman nursing home resident is brought to the emergency department by ambulance because of seizures and loss of consciousness. She has been drowsy, agitated and "shaking" since yesterday. Seizures ceased after intravenous midazolam 2.5 mg was administered. She was endotracheally intubated, given normal saline infusion, and admitted to the intensive care unit. Her vital signs are temperature 38 °C, heart rate 110 beats/min, and blood pressure 150/80 mm Hg. She has horizontal nystagmus, brisk deep tendon reflexes, and sustained clonus (eight beats) bilaterally. Her serum lithium concentration is 4.5 mEq/L with a creatinine of 3.0 mg/dL. You appropriately decide that an extracorporeal technique is needed to treat her lithium toxicity. Which of the following is the most appropriate choice? A. Continuous venovenous hemodialysis (CVVHD) B. Intermittent hemodialysis (IHD) C. Sustained low-efficiency dial

The correct answer is B. Rationale: This patient is severely lithium toxic, and her lithium body burden should be reduced urgently. Intermittent hemodialysis (IHD) is the most efficient method for rapidly removing lithium from the central compartment, achieving clearance rates of up to 170 to 180 mL per minute (choice B is correct). This patient has stable hemodynamic parameters, so she should be able to tolerate IHD. In the event that IHD is unavailable or when the patient is unstable, other extracorporeal techniques may be considered (choices A, C, and D are not the best answers).

A 30-year-old man is admitted to the hospital with hallucinations. He reports self-medicating with excessive amounts of clonazepam daily for several months with his last dose more than 24 hours ago. His vital signs are HR 110 beats/min, BP 150/90 mm Hg, RR 18 breaths/min, T 37 °C. Which one of the following is the most appropriate initial treatment? A. Clonidine B. Diazepam C. Propranolol D. Sodium oxybate

The correct answer is B. Rationale: This patient's history and clinical presentation is consistent with benzodiazepine withdrawal. The best course of action is to administer a long- acting benzodiazepine and slowly taper dosage over 2 to 4 weeks, as withdrawal symptoms can be protracted. Tapering allows a slow decrease in tolerance without severe withdrawal symptoms. Alternatively, a similar cross-tolerant agent can be used. Long-acting benzodiazepines such as diazepam or chlordiazepoxide are preferred for severe withdrawal (choice B is correct). Short-acting agents may be disadvantageous because maintenance of therapeutic serum drug concentrations requires frequent dosing. In patients with moderate- to-severe symptoms (eg, seizures and delirium), small IV boluses (eg, diazepam 5 mg) should be administered until adequate sedation is achieved. Patients experiencing milder symptoms can be treated via the oral route. Barbiturates such as pentobarbital and phenobarbital can also be used for the treatment of benzodiazepine withdrawal. β-Blockers and clonidine have been used to treat benzodiazepine withdrawal. Propranolol (10-40 mg every 6 hours) may ameliorate tremor, muscle twitching, tachycardia, and hypertension. However, it has little effect on anxiety, agitation, and dysphoria (choice C is incorrect in this case). Similarly, while clonidine has been advocated, its effectiveness for modulating the intensity, severity, and duration of withdrawal is unclear (choice A is incorrect). As with ethanol withdrawal, it is important to realize that treating peripheral manifestations of withdrawal may obscure early signs of impending delirium tremens, and impedes the assessment of adequate sedation. For this patient with likely benzodiazepine use disorder, resuming a short-acting benzodiazepine in the absence of other treatment

A 65-year-old man with a history of depression and paroxysmal atrial fibrillation presents by ambulance for evaluation of bradycardia and altered mental status after a suicide attempt. The patient ingested an unknown amount of his home flecainide 4 hours ago. He had a heart rate in the 30s, blood pressure 80/40 mm Hg, and minimal responsiveness to painful stimuli; therefore, he was intubated in the field for airway protection and started on intravenous fluids. On arrival to the emergency department, his vital signs are heart rate 40 beats/minute, blood pressure 110/60 mm Hg, and temperature of 37°C. An ECG confirms sinus bradycardia, with QRS interval of 140 ms, and QTc 460 ms. What is the most appropriate next step in management? A. Hemodialysis B. Hypertonic sodium bicarbonate C. Intravenous magnesium D. Temporary cardiac pacing

The correct answer is B. Rationale: This patient's prolonged QRS of 140 ms should narrow with sodium bicarbonate, which has been reported to be effective for flecainide overdose (choice B is correct). The goal with this therapy is to provide enough sodium to compete with flecainide's mechanism of action of inhibiting fast sodium channels. Hemodialysis is unlikely to be effective given Flecainide has high protein binding and lipid soluble (choice A is incorrect). Magnesium is not indicated as this patient has a normal QTc (choice C is incorrect). Additionally, class IC agents have not been shown to cause torsades de pointes. While this patient's heart rate is low, he is not profoundly hypotensive, and has been fluid responsive. Temporary cardiac pacing may be needed eventually, but it is not the best next step (choice D is incorrect).

A 43-year-old man presents 12 hours after an acetylsalicylic acid (Aspirin) overdose. His initial salicylate concentration is 65 mg/dL and he is started on a bicarbonate drip at twice maintenance fluids rate with potassium supplementation. His blood gas reveals a serum pH of 7.5. Potassium concentration is 4.5 mmol/L with a normal creatinine. Urine pH is 8 and his urine output is 2 cc/kg/h. The patient's mental status is normal and respiratory status is stable, as demonstrated by a persistent respiratory alkalosis. Subsequent salicylate concentrations obtained every 2 hours are 68 mg/dL, 66 mg/dL, and 75 mg/dL. What is the most appropriate next step in management? A. Acetazolamide B. Activated charcoal C. Hemodialysis D. Intubation and hyperventilation

The correct answer is B. Rationale: This patient's serial salicylate concentrations are most likely explained by ongoing absorption of acetylsalicylic acid, potentially due to delayed gastric emptying and bezoar formation. Activated charcoal, even several hours after acute overdose, would be expected to adsorb drug remaining in the stomach and thus decrease its subsequent absorption from the small intestine (choice B is correct). Acetazolamide causes systemic acidosis and is contraindicated in salicylate overdose (choice A is incorrect). The patient is ventilating well spontaneously, and his treatment has reached the medically optimal targets for urine output, urine pH and blood pH. He may eventually require hemodialysis if his salicylate concentration continues to rise, but at this time he is clinically well and has not failed standard therapy, but for the lack of GI decontamination (choice C is incorrect). Mechanical ventilation is not indicated and could result in a life-threatening drop in pH when sedated or paralyzed for intubation due to loss of his natural hyperventilation (choice D is incorrect).

A 25-year -old woman presents to the emergency room hallucinating after attending a religious ceremony. She reports a massive amount of vomiting prior to the hallucinations. Vital signs on arrival are blood pressure of 120/60 mm Hg, heart rate 95 beats/minute, respiratory rate 16 breaths/minute, and temperature 36 °C. Her SpO2 is 98% on room air and finger stick blood glucose is 100 mg/dL. Which of the following was most likely ingested during the ceremony? A. Hawaiian baby woodrose B. Morning glory C. Peyote D. Psilocybin

The correct answer is C. Rationale: Peyote often causes profound nausea and vomiting followed by hallucinations (choice C is correct). While all of the other answer choices are also hallucinogens, none of them regularly cause severe nausea and vomiting prior.

A 52-year-old man presents to the emergency department complaining of headache, dizziness, and nausea. Symptoms developed after spraying a pesticide 1 hour prior to arrival. He was wearing a respirator while spraying. He is ambulatory to an examination room. Vital signs and physical examinations are normal; he does not have any increased oral secretions, lacrimation, or rhinorrhea. An intravenous line is established, and an antiemetic medication is administered. What is the most appropriate next step for the management of this patient? A. Atropine B. Decontamination C. Physostigmine D. Glycopyrrolate E. Reassurance and discharge home

The correct answer is B. Rationale: This presentation is consistent with mild organophosphate poisoning onset after spraying pesticides. After an initial assessment, decontamination should be performed (choice B is correct). All the patients clothing should be removed, bagged, and discarded. Porous items, such as leather, should be discarded as decontamination of these products is very difficult. Jewelry should also be removed, bagged, and not returned until thoroughly washed. The patient's body should be washed with mild soap and water. Hospital staff require proper personal protective equipment during this stage. Dermal exposure can lead to delayed systemic effects, emphasizing the importance of proper decontamination. Cholinergic signs rarely present after 12 hours unless exposure to a highly lipophilic compound has ocurred. Progressive or prolonged symptoms should raise suspicion of continued absorption or exposure to the poison. Atropine may be required if the patient develops acute cholinergic crisis, but not indicated now given lack of bronchorrhea, bronchoconstriction, bradycardia, or hypotension (choice A is incorrect). Physostigmine is a reversible acetylcholinesterase inhibitor that increases the acetylcholine levels and is the treatment for antimuscarinic delirium (choice C is incorrect). Glycopyrrolate is an antimuscarinic agent that does not penetrate the central nervous syndrome (choice D is incorrect). It can be substituted for atropine when isolated peripheral cholinergic toxicity is present. Reassurance and discharge are not appropriate because the patient should be observed 12 to 24 hours for progressive signs and symptoms of organophosphate poisoning (choice E is incorrect).

What is the most common cause of hydrocarbon-associated fatalities? A. Acute renal tubular necrosis B. Bone marrow suppression C. Cardiac dysrhythmias D. Fulminant hepatic failure

The correct answer is C. Rationale: Acute exposures to hydrocarbon gases and vapors can affect the eyes, lungs, and central nervous system. If present in sufficient concentrations to displace oxygen, this exposure can sensitize the heart to catecholamines, causing abnormal rhythms and ventricular fibrillation leading to sudden death (choice C is the correct answer because choices A, C, and D are less common). Even a 30 second exposure to high concentrations of hydrocarbons and a low-oxygen atmosphere can result in the rapid onset of respiratory depression, hypoxia, and fatal cardiac arrhythmias. Preexisting coronary artery disease may exacerbate that risk.

Which one of the following statements is correct regarding the use of flumazenil to reverse sedation following a drug overdose? A. It is contraindicated in patients aged less than 12 years B. It is safe to use in patients on long-term benzodiazepine therapy C. It should be avoided in patients with suspected multidrug overdose D. Its use reduces intensive care unit admission rates

The correct answer is C. Rationale: Flumazenil should be avoided for patients suspected of multidrug overdose, as there is potential to unmask the epileptogenic effects of coingested substances (choice C is correct). One of the potential indications for flumazenil use may be to reverse the effects of benzodiazepine ingestions by toddlers (choice A is incorrect). For patients on long-term benzodiazepine therapy, there is a risk of precipitating withdrawal and seizures with flumazenil (choice B is incorrect). The use of flumazenil has not been demonstrated to reduce hospital length of stay or the need for high-intensity monitoring for patients with benzodiazepine overdose (choice D is incorrect).

Which of the following laboratory findings correlate with the degree of acute theophylline poisoning? A. Hypocalcemia and hypophosphatemia B. Hypochloremia and metabolic alkalosis C. Hypokalemia and hyperglycemia D. Hyponatremia and hypomagnesemia

The correct answer is C. Rationale: Hypokalemia and hyperglycemia strongly correlate with the degree of intoxication from acute theophylline poisoning (choice C is correct). Other metabolic disturbances accompanying theophylline intoxication include hypophosphatemia, hypomagnesemia, and hypercalcemia are not as tightly associated with theophylline levels (choices A and D are not the best answers). Respiratory alkalosis is the most common acid-base alteration, but mild metabolic acidosis has been reported (choice B is incorrect).

Iron toxicity is well known to directly damage all of the following organs, EXCEPT: A. Heart B. Intestine C. Kidney D. Liver

The correct answer is C. Rationale: Iron is not directly nephrotoxic (choice C is the "correct", wrong answer). However, acute renal failure may result from shock due to fluid and blood loss, or even from the deferoxamine given for chelation, especially in the absence of adequate fluid replacement. Hemorrhagic necrosis of gastrointestinal mucosa can occur. At the cellular level, iron impairs metabolism in the heart and liver.

Which one of the following is the preferred initial method for enhancing phenobarbital elimination following an overdose? A. Charcoal hemoperfusion B. Continuous renal replacement therapy C. Multidose activated charcoal D. Urinary alkalinization

The correct answer is C. Rationale: Multidose activated charcoal (MDAC) is the preferred initial method for enhancing phenobarbital elimination and has been shown to be more effective than urinary alkalinization (choice C is correct, choice D is incorrect). Extracorporeal elimination methods can be employed if MDAC is failing, and the preferred extracorporeal technique is intermittent hemodialysis (choices A and B are incorrect).

A 22-year-old woman with HIV, tuberculosis, and depression who recently immigrated from Bangladesh is brought to the hospital in status epilepticus. On arrival her vital signs are heart rate 120 beats per minute, blood pressure150/90 mmHg, respirations 20 per minute, and temperature 38 °C. Initial laboratory studies with normal values in parentheses are shown: Serum sodium: 138 mEq/L (136-144 mEq/L). Serum potassium: 5.8 mEq/L (3.7-5.2 mEq/L). Serum chloride: 99 mEq/L (96-106 mEq/L). CO2 (carbon dioxide): 13 mEq/L (22-28 mEq/L). BUN: 11 mg/dL (8-23 mg/L). Creatinine: 0.9 mg/dL (0.3-0.9 mg/L).Glucose: 220 mg/dL (70-110 mg/dL).Serum calcium (total): 9.1 mg/dL (8.2-10.2 mg/dL). Serum AST (SGOT) 60 IU/L (8-48 IU/L).Serum ALT (SGPT) 49 IU/L (7-55 IU/L). Treatment with 4 mg intravenous lorazepam three times over 30 minutes is ineffective for stopping her seizures. An overdose of which of the following patient's medicati

The correct answer is C. Rationale: Of the drugs listed, this clinical picture of seizures refractory to GABAA receptor agonist lorazepam, coma, and metabolic acidosis would be due to isoniazid overdose (choice C is correct). None of the other medication choices cause seizures (choices A, B, and D are incorrect). Overdoses of dolutegravir and lamivudine result in lactic acidosis, hypokalemia, and altered mental status. Somnolence, nausea, vomiting, tachycardia, QT prolongation, and tremor occur with escitalopram. Nausea, vomiting, lethargy, and hepatoxicity may occur after rifapentine overdose.

You have successfully treated a patient with agitated anticholinergic delirium using physostigmine. However, careful monitoring of the patient is required to observe for which one of the following consequences of physostigmine treatment? A. Bradycardia B. Bronchospasm C. Recurrent sedation D. Seizures

The correct answer is C. Rationale: Physostigmine has a short half-life relative to the drugs that cause anticholinergic delirium and therefore recurrence of the delirium and depressed mental status is common and readministration may be needed (choice C is correct). Note that anticholinergic agents slow gastrointestinal motility and that physostigmine administration may stimulate gut motility enhancing the absorption of drug remaining in the GI tract, actually increasing concentrations of the anticholinergic drug. Bradycardia and seizures are the result of cholinergic stimulation and usually occur around of the time of physostigmine administration (choices A and D are incorrect). Bronchospasm would be a contraindication to initial physostigmine administration as the cholinergic stimulation may exacerbate the bronchospasm (choice B is incorrect).

A 3-year-old child is found drinking kerosene in the garage. She initially coughed but has been asymptomatic ever since. The parents decide to have her evaluated and arrive at the emergency department one hour later, at which time she still has no symptoms. What is the most appropriate management? A. Administer activated charcoal B. Admit for overnight observation C. Check chest x-ray in 3 to 5 hours; if normal discharge to home D. Discharge to home with continued monitoring by parents

The correct answer is C. Rationale: Pneumonitis may be seen on x-ray as early as 2 hours after poisoning. If there are no symptoms, and if the chest film is normal 4 to 6 hours after ingestion, then the patient can be discharged (choice C is correct). It would be premature to discharge home or admit without imaging (choices B and D are incorrect). Of note, activated charcoal is specifically contraindicated in hydrocarbon poisoning, as it does not bind hydrocarbons and it increases the risk of hydrocarbon aspiration (choice A is incorrect).

A 25-year-old woman is admitted to an intensive care unit soon after an intentional β-blocker overdose. She is alert and answering questions. Her heart rate is 116/min and her blood pressure is 150/100 mm Hg. Laboratory results including serum electrolytes are normal. An electrocardiogram shows PR 200 ms, QRS 140 ms, and QTc 450 ms. She acutely becomes stuporous and has a generalized seizure lasting several minutes. Which one of the following β- blockers is most likely involved in this overdose? A. Carvedilol B. Labetolol C. Propranolol D. Sotalol

The correct answer is C. Rationale: Propranolol, the most lipophilic β-blocker, can easily cross the blood-brain barrier and cause seizures in overdose cases (choice C is correct). Some β-blockers such as propranolol also block cardiac sodium channels producing membrane-stabilizing effects and QRS prolongation, while others such as sotalol block potassium channels, causing QTc prolongation (choices A, B, and D are incorrect).

A 78-year-old woman with dementia receives digoxin-specific antibody fragments after an accidental digoxin overdose and is admitted to the intensive care unit. In addition to continued cardiac telemetry, which of the following is a potential treatment complication to monitor for over the next 24 hours? A. Anaphylaxis B. Hypertension C. Hypokalemia D. Rebounding digoxin levels

The correct answer is C. Rationale: Recurrent toxicity and dysrhythmias can be seen in 3% of patients after digoxin-specific antibody fragments (Fab) therapy, so ongoing cardiac telemetry is necessary. A precipitous drop of potassium levels after treatment is a known complication (choice C is correct). Like most dysrhythmias, tight control of serum electrolytes within normal ranges should be sought. There are no reports of anaphylaxis from digoxin-specific antibody fragments therapy though more mild drug reactions have been reported (choice A is incorrect). Hypertension is also not a recognized adverse effect (choice B is incorrect). Serum digoxin concentration is unreliable for up to several days after Fab and an increase or rebound in digoxin level is not an indication of toxicity (choice D is incorrect).

A 75-year-old man with a history of coronary artery disease (CAD) is admitted for a total hip arthroplasty. During the procedure, his vital signs remain within normal limits, although he experiences significant bleeding. In the recovery room, he is found to have a hematocrit of 26%, so a 2-unit RBC transfusion is ordered. Halfway through administration of the second unit of blood, the patient begins to cough and complains of tightness in his chest and a severe headache. He is afebrile, but his other vitals are altered, as follows: Pre: Blood Pressure 120/75 mm Hg post: 165/85 mm Hg Pre: Heart Rate 78 bpm post: 120 bpm Pre: Respiratory Rate 20 bpm post: 32 bpm The patient is treated with supplemental oxygen and a diuretic. Approximately 35 minutes after these interventions, he feels much better. What was the most likely blood transfusion reaction experienced? A. Acute hemolytic transfusion reaction (AHTR) B. Anaphyl

The correct answer is C. Rationale: The differential diagnosis of acute respiratory distress accompanying transfusion includes TACO, TRALI, and anaphylaxis. Patients with a diminished cardiac reserve and/or significant anemia who receive blood too quickly are at a risk for acute pulmonary edema secondary to congestive heart failure (ie, TACO; choice C is correct). TACO is the leading cause of transfusion-associated fatalities reported to the FDA over the 5-year reporting period: FY2015 to FY2019. AHTR and transfusion-associated sepsis would typically present very differently (eg, fever, hypotension, lack of pulmonary signs and symptoms) (choice A is incorrect). The increased blood pressure and pulmonary edema with no rash make the diagnosis of anaphylaxis less likely (choice B is incorrect). His reaction is unlikely to be TRALI because he experienced hypertension (rather than hypotension), was afebrile, and responded favorably to diuretic therapy (choice D is incorrect).

A 50-year-old man presents to the emergency department after feeling unwell for the past few days. He had significant diarrhea and vomiting, fatigue, nausea, and difficulty walking. He continued to take his medications, including lithium 750 mg and perindopril 8 mg daily. On physical examination, he is alert and oriented with normal vital signs. Pulmonary, cardiac, and abdominal examinations are unremarkable. He has a mild tremor and hyperreflexia in both lower extremities, but no sustained clonus. Laboratory investigations 12 hours after his last lithium dose showed serum lithium concentration 2.5 mEq/L, creatinine 2 mg/dL, BUN 40 mg/dL, Na 135 mEq/L, K 5 mEq/L, Cl 96 mEq/L, and HCO3 26 mEq/L. What is the most appropriate next step in management? A. Forced diuresis B. Hemodialysis C. Normal saline infusion D. Sodium polystyrene sulfonate

The correct answer is C. Rationale: This patient has mild lithium toxicity as evidenced by fatigue, nausea, tremor, and hyperreflexia. He developed acute kidney injury with prerenal azotemia because of fluid and sodium loss from the diarrhea and vomiting. More than 95% of absorbed lithium is excreted by the kidneys. Sodium depletion can result in as much as a 50% increase in lithium reabsorption, so intravascular fluid replacement or resuscitation with normal saline is the most important treatment for this patient (choice C is correct). Forced diuresis (eg, administering loop diuretics or osmotic agents) may initially increase elimination, but has the potential to cause volume depletion and sodium loss (choice A is incorrect). Hemodialysis can be considered for a patient with central nervous system manifestations of lithium toxicity (eg, confusion, irritability, ataxia, seizures) and or kidney impairment, but intravenous normal saline infusion should be tried first (choice B is not the best answer). Sodium polystyrene sulfonate or Kayexalate exchanges sodium for lithium and may enhance its elimination based on animal studies and healthy volunteers, but it is a relatively slow process and not as beneficial as acutely administering normal saline (choice D is incorrect). Moreover, sodium polystyrene sulfonate use is associated with hypokalemia, constipation, volume overload, and colonic necrosis.

A 68-year-old previously healthy man presents with weakness, fatigue, and lethargy. He has noticed dark urine and yellowing of his eyes over the last week. He is not taking any medications and denies use of herbal supplements. Laboratory data are shown with normal ranges in parentheses: Hemoglobin 7.9 g/dL (12-16 g/dL) Hematocrit 24% (36%-48%) MCV 107 fL (80-100 fL) WBC 6.0 K/μL (4.5-10.5 K/μL) Platelet count 154 K/μL (150-450 K/μL) Reticulocyte count 9% (0.5%-1.8%) LDH 894 U/L (<200 U/L) Total bilirubin 3.9 mg/dL (<1.2 mg/dL) Direct bilirubin 0.9mg/dL (<0.3 mg/dL) Coombs test: IgG Strongly positive (negative) Coombs test: C3 Negative (negative) Which of the following do you expect to see when you review the blood smear? A. Bite cells B. Schistocytes C. Spherocytes D. Target cells

The correct answer is C. Rationale: The history and labs are consistent with a hemolytic anemia. The Coombs, strongly positivity for IgG, is consistent with warm autoimmune hemolytic anemia and spherocytes would be expected on the blood smear (choice C is correct). Target cells would be seen in thalassemia, and bite cell in G6PD deficiency, both of which are Coombs negative (choices D and A are incorrect). Schistocytes would be expected in thrombotic thrombocytopenia purpura (TTP) and other microangiopathic anemias, which would be Coombs negative (choice B is incorrect). Although not listed here as a choice, RBC agglutination would be seen in cold agglutinin disease and the Coombs would be IgG negative and C3 positive.

A 42-year-old man with no known past medical history presented to the emergency department with acute cholinergic crisis after exposure to an organophosphorus agent. He was satisfactorily treated with atropine and pralidoxime; an atropine and pralidoxime infusion was initiated and the patient was admitted to the intensive care unit. Vital signs: BP 128/70 mm Hg, HR 105 beats/min, RR 20 breaths/min, T 37.5 °C, and pulse oximetry is 100% on room air 36 hours after presentation. The patient has recovered and is free of muscle fasciculations. Atropine and pralidoxime infusions are discontinued. Cardiac and pulmonary examinations are unremarkable. However, the patient is noted to have difficulty lifting his head off the bed while talking to you and complains that his arms "feel heavy." What is the most likely diagnosis? A. Chronic organophosphorus-induced neuropsychiatric disorder (COPIND) B. Critical illness polyneu

The correct answer is C. Rationale: The patient described is developing intermediate syndrome (IMS) (choice C is correct). The onset of IMS is between 24 and 96 hours following exposure to an organophosphorus agent. It is characterized by weakness of neck muscles, proximal limb muscles, and muscles of respiration. The first sign is often inability to lift head from bed. Patients must be closely monitored for paralysis of muscles involved in respiration. Organophosphate-induced delayed polyneuropathy (OPIDP) is an axonopathy occurring 1 to 3 weeks after cholinergic crises. OPIDP is a sensorimotor peripheral neuropathy that is characterized by both paresthesias and motor dysfunction occurring first in the longest skeletal nerves with development of foot drop and a high-stepping gait. Symptoms slowly develop and can be divided into three phases: progressive, stationary, and improvement. During the progressive phase, patients have a peripheral sensory neuropathy with complaints of burning, tightness, or pain in the legs and feet. This is followed by numbness and tingling. Subsequently, motor weakness develops, with weakness and loss of peroneal innervation causing foot drop. After approximately 1 week, the paresis may symmetrically ascend into the upper extremities. The sensory loss may occur in a stocking-glove distribution, and the patient loses proprioception. With time, a positive Romberg sign and loss of lower extremity deep tendon reflexes may develop. Flaccid paralysis may occur in severe cases. During the stationary phase, paresis and motor findings cease to progress. This may occur over 3 to 12 months. The improvement phase usually begins 6 to 18 months after exposure, though improvement may be seen as early as 2 to 9 weeks. Partial or complete motor function returns in reverse order of loss. During this phase, ce

A 39-year-old man presents to the emergency department with nausea, vomiting, diarrhea, abdominal pain, and dark-colored urine that has persisted for the past 12 hours. He has been generally healthy aside from an episode of acute appendicitis 6 years ago that resulted in an appendectomy. His social history is notable for occasional ethanol consumption (he reports 2-3 beers per month). He works as a smelter. On physical examination, his heart rate is 114 beats per minute and the remainder of his physical examination is normal. He is uncomfortable, intermittently retching, with a dusky, bronze tint to his skin. His abdomen is soft, non-distended, nontender, and without organomegaly. A complete blood count is notable for a hemoglobin of 8.6 g/dL, and a blood smear demonstrates anisocytosis and poikilocytosis. Which one of the following treatment options is the most appropriate for this patient? A. British anti-Lewisite

The correct answer is C. Rationale: The patient's clinical presentation of abdominal pain and bronze skin, along with laboratory evidence of hemolysis, in a smelter worker, is consistent with arsine toxicity. Of the treatment options, only exchange transfusion is an appropriate therapeutic intervention (choice C is correct). British anti-Lewisite has not been shown to be beneficial in cases of acute hemolytic anemia due to arsine toxicity, though it is used in other forms of arsenic poisoning (choice A is incorrect). Calcium disodium edetate is appropriate for the treatment of lead poisoning, not arsine poisoning (choice B is incorrect). 2,3-Dimercaptosuccinic acid is used for mercury poisoning or less severe poisoning from lead or other forms of arsenic but would be inappropriate treatment for arsine gas poisoning (choice D is incorrect).

A 72-year-old man with a history of mild dementia with agitation, coronary artery disease, hyperlipidemia, non-insulin-dependent diabetes mellitus, and peripheral artery disease is brought in from home after consuming 1 week's worth of his medications. Those medications include lisinopril, atorvastatin, metformin, and quetiapine. His heart rate is 94 beats/min, blood pressure 134/78 mm Hg, respiratory rate 18 breaths/min, temperature 36.8 °C, oxygen saturation 94% on room air, and finger stick glucose of 140 mg/dL. His examination reveals him to be awake and alert. He is oriented to person, place, and year. He's not sure why he's in the emergency department. His physical exam is entirely normal. An ECG demonstrates a rate of 94, PR of 160 ms, QRS of 96 ms, and QTc of 400 ms. A urine drug of abuse test is reflexively sent by the nurse, and it results as positive for tricyclic antidepressants (TCAs). What is t

The correct answer is C. Rationale: This clinically stable patient is best served by continued monitoring and support (choice C is correct). Quetiapine is well known to cause false-positive urine TCA drug testing results. The patient's medication list does not include a TCA, and his physical examination is not consistent with TCA effects. Therefore, anti-TCA monoclonal antibody infusion or intralipid treatments are not indicated (choices A and D are incorrect). There is also no indication for benzodiazepine therapy at this point in time (choice B is incorrect).

A 44-year-old woman presents after an overdose of haloperidol. Her vital signs are heart rate 105 beats/min, blood pressure 116/58 mm Hg, respiratory rate of 20 breaths/minute, temperature 37.2 °C, and oxygen saturation 97% on room air. Her mental status appears normal. Pupils are 4 mm bilaterally and reactive. Her skin is warm with some axillary sweat present. Reflexes are 2+ symmetrically. An ECG demonstrates PR of 180 ms, QRS of 106 ms, and QTc of 610 ms. What is the most appropriate next step in management? A. Amiodarone 150 mg bolus followed by continuous infusion B. Hypertonic saline C. Magnesium IV D. Oral propafenone

The correct answer is C. Rationale: This patient has an increased QT interval due to haloperidol. Initial treatment of prolonged QT interval in a stable patient is the administration of IV magnesium (choice C is correct). Other electrolytes should also be normalized, particularly potassium. In the absence of torsade de pointes, there is no indication for cardiac pacing. Neither amiodarone nor propafenone have a role during the treatment of prolonged QT in the absence of a ventricular dysrhythmia (choices A and D are incorrect). Hypertonic saline also has no role for the treatment of this patient in the absence of a marked QRS prolongation (choice B is incorrect).

Which of the following are appropriate treatments for severe calcium channel blocker (CCB) toxicity? A. High-dose insulin and glucose B. Intravenous calcium chloride C. Oral activated charcoal D. All of the above

The correct answer is D. Rationale: There is no specific antidote for CCB poisoning. Patients severely poisoned will need multiple concurrent therapies, often at doses much higher than typically used (choice D is correct). Not listed here as a choice, but vasopressor support is also usually required.

An 80-year-old woman with arthritis presents with delirium following increased use of acetylsalicylic acid orally and methylsalicylic acid topically. Her serum salicylate concentration is 50 mg/dL and arterial blood gas reveals a pH of 7.30 and pCO2 of 40 mm/Hg. She is started on a bicarbonate infusion at twice the maintenance fluids rate with potassium supplementation. Her mental status deteriorates with intermittent agitation and increasing delirium despite boluses of D50W. What is the most appropriate next step in management? A. Activated charcoal B. Continued observation C. Hemodialysis D. Lorazepam

The correct answer is C. Rationale: This patient requires hemodialysis given her progression to severe salicylate toxicity despite treatment (choice C is correct). She is likely unable to generate the respiratory alkalosis expected with salicylate overdose, and has an altered mental status indicating a high end-organ (brain) burden of salicylate. Emergency hemodialysis is indicated to rapidly draw salicylate out of the central nervous system. While the patient's salicylate level is not extremely high, the severity of her symptoms, the chronicity of the overdose, and the continued worsening despite medical therapy—especially of her mental status—all make further observation without hemodialysis very high risk (choice B is incorrect). Activated charcoal is unlikely to significantly help with chronic toxicity and would increase the risk of aspiration for this altered patient (choice A is incorrect). She has been appropriately empirically treated with supplemental dextrose for possible occult CNS hypoglycemia without improvement. This patient should not be sedated as it would be expected to further decrease her respirations and worsen her acidosis (choice D is incorrect).

For a symptomatic patient who is a "body packer" and has ingested cocaine, which of the following is the most appropriate therapy in addition to supportive care? A. Activated charcoal B. Cathartics C. Endoscopic packet removal D. Surgical exploration

The correct answer is D. Rationale: "Body packers," first described in 1973, are persons who, voluntarily or through coercion, swallow or insert drug-filled packets into a body cavity for smuggling purposes. Management depends on many factors to include the ingested substance. While asymptomatic patients may be monitored and given whole-bowel irritation, symptomatic sympathomimetic toxicity (cocaine or amphetamines) should be treated with surgical removal of the packets (choice D is correct; choices A, B, and C are not the best answers). Endoscopic removal has been reported but risks packet rupture.

A 60-year-old man presents to the emergency department following an intentional lithium SR (sustained release) overdose of 9 g one hour before arrival. He has been on lithium SR 450 mg twice daily for 10 years due to bipolar disorder. His serum lithium level was in therapeutic range 1 month ago. He is awake and alert with normal vital signs and has active bowel sounds. What is the most appropriate approach to gastrointestinal decontamination? A. Activated charcoal B. Gastric lavage C. Sodium polystyrene sulfonate D. Whole-bowel irrigation

The correct answer is D. Rationale: Because he ingested a large amount of lithium sustained release, has presented to the hospital early, and has no contraindications, this patient may benefit from GI decontamination. He is at high risk of neurologic toxicity because he already has a significant central nervous system lithium burden from chronic use. Whole bowel irrigation is the most appropriate method (choice D is correct). Activated charcoal does not bind with lithium (choice A is incorrect). The role of gastric lavage for removing modified released preparation from the stomach is unclear (choice B is not the best answer). Sodium polystyrene sulfonate involves a slow cation exchange process and is inappropriate for urgent GI decontamination (choice C is incorrect).

Which acid-base disturbances(s) would be consistent with salicylate overdose? A. Respiratory acidosis with metabolic acidosis B. Respiratory alkalosis C. Respiratory alkalosis with metabolic acidosis. D. All of the above

The correct answer is D. Rationale: Early overdose is associated with direct stimulation of the respiratory center leading to primary respiratory alkalosis. As overdose progresses, a coprimary metabolic acidosis develops while the respiratory alkalosis is still present. In late overdose, confusion, fatigue, seizures, cerebral edema, and mechanical ventilation can all contribute to respiratory acidosis, which in turn causes a rapid fall in blood pH facilitating access of salicylate into the brain—a rapid downward spiral (choice D is the best answer).

Where is most ingested ethanol absorbed? A. Colon B. Duodenum C. Jejunum D. Stomach

The correct answer is D. Rationale: Ethanol is readily absorbed from the gastrointestinal tract, with about 70% occurring in the stomach and 25% within the duodenum. Peak ethanol levels typically occur 30 to 60 minutes after ingestion. This makes gastric lavage or use of activated charcoal usually avoided unless coingestion is suspected, as it is largely ineffective if not done right after ingestion.

A 37-year-old woman presents with bilateral hand pain. She states that she was using a rust removal product and was not wearing gloves. She stopped using the product around 6 hours ago and pain developed 2 hours prior to presentation. Her vital signs are unremarkable and exam is normal other than some redness of the hands. After irrigation with water, what is the most appropriate next step for management? A. Intra-arterial calcium gluconate B. Regional intravenous administration (Bier block) of calcium gluconate C. Subcutaneous injection of calcium gluconate D. Topical calcium gluconate

The correct answer is D. Rationale: Initial conservative management with topical treatment is effective for most patients (choice D is correct). Patients who fail topical treatment should be considered for the more aggressive measures as listed.

By what mechanism does pyridoxine serve as an antidote for isoniazid overdose? A. It antagonizes glutamate receptors B. It enhances acetalization of isoniazid C. It inhibits conversion of isoniazid into hydrazine D. It promotes the synthesis of GABA

The correct answer is D. Rationale: Isoniazid interferes with the pyridoxine-mediated conversion of glutamate to γ-aminobutyric acid (GABA). Administration of pyridoxine allows it to be converted to pyridoxyl 5′ phosphate which is required in the glutamic oxaloacetic transaminase-mediated conversion of glutamate to GABA (choice D is correct). Pyridoxine by itself has no effect on the metabolism of isoniazid or its metabolites, and no direct effect on neuronal receptors (Choices A, B, and C are incorrect).

A 46-year-old man with a history of cirrhosis and oxygen-dependent chronic obstructive lung disease presents to the emergency department after a mechanical fall resulting in a left hip fracture. While in the emergency department, he receives a nerve block with multiple doses of bupivacaine. Shortly after the procedure, he becomes unresponsive, does not have a palpable pulse, but there is a regular narrow rhythm on the monitor. Cardiopulmonary resuscitation is started. Which of the following medications is most likely to lead to return of spontaneous circulation? A. Amiodarone B. Epinephrine C. Intravenous magnesium D. Lipid emulsion

The correct answer is D. Rationale: Lipid emulsion (intralipid) is the standard treatment for bupivacaine-induced cardiac arrest (choice D is correct). It is thought that the intralipid acts by partitioning bupivacaine away from the cardiac receptors into an intravenous lipid phase. Excellent animal evidence and several human case reports have shown it to be effective. This patient is in pulseless electrical activity (PEA) and not ventricular fibrillation or ventricular tachycardia, and thus amiodarone is not indicated (choice A is incorrect). While epinephrine is standard of care in Advanced Cardiac Life Support (ACLS), it is unlikely to be effective here given the patient has suffered an overdose of bupivacaine (choice B is incorrect). Bupivacaine would be unlikely to prolong the QT interval. Additionally, this patient was in pulseless electrical activity, not torsades de pointes, so magnesium would not be helpful (choice C is incorrect).

An 18-month-old child presents after placing a bottle of 5% hydrofluoric acid rust remover to her mouth. It is not clear if she ingested any. She is asymptomatic and has a normal exam. What is the most appropriate management? A. Empiric administration of IV calcium B. Gastric lavage C. Oral administration of calcium carbonate D. Treatment only if hypocalcemia develops

The correct answer is D. Rationale: Most accidental pediatric ingestions do well without treatment (choices A, B, and C are not the best answers). Given that the amount ingested is not clear, it is reasonable to monitor and treat if hypocalcemia develops (choice D is correct).

Physostigmine may treat delirium caused by which of the following drugs? A. Dexamethasone B. Diazepam C. Methamphetamine D. Olanzapine

The correct answer is D. Rationale: Olanzapine is an atypical antipsychotic with anticholinergic properties that the anticholinesterase inhibitor physostigmine can reverse (choice D is correct). Diazepam and phenobarbital produce CNS depression that may include delirium, but an acetylcholinesterase inhibitor will not reverse delirium caused by these drugs since they act upon the GABA receptor (choice B is incorrect). Dexamethasone may produce psychosis, but it would not be treated by physostigmine (choice A is incorrect). Methamphetamine-induced delirium is treated with medications such as benzodiazepines and antipsychotics (choice C is incorrect).

An unresponsive 75-year-old woman with a history of hypertension, diabetes mellitus, and atrial fibrillation is brought in to the emergency department. A CT scan shows intracerebral hemorrhage. She has been on long- term anticoagulation with warfarin for the atrial fibrillation and her INR is 10.5. What is the most appropriate management? A. Cryoprecipitate B. Fresh frozen plasma (FFP) C. Intravenous (IV) vitamin K D. Prothrombin complex concentrate (PCC)

The correct answer is D. Rationale: PCC is the best choice for warfarin reversal in this patient (choice D is correct). When there is major life-threatening bleeding such as intracranial hemorrhage, the patient requires rapid and complete reversal of warfarin. PCCs, in combination with intravenous vitamin K, will reverse warfarin within 15 minutes. If PCCs are not available, fresh frozen plasma may be used. However, plasma requires a large volume of administration, often delaying the time required to reverse the INR.

A 23-year-old man was admitted with alcohol and ethylene glycol intoxication requiring intubation, mechanical ventilation, sodium bicarbonate, and fomepizole. After a couple of days, he was improving but then appeared to have severe alcohol withdrawal, treated with thiamine and high-dose diazepam infusion. You are called in on hospital day 6 because he has hyperosmolarity, with an increasing anion gap, and return of renal failure. What is the most appropriate next step in management? A. Continue supportive care B. Ethanol infusion C. Repeat course of fomepizole D. Stop diazepam, consider dialysis

The correct answer is D. Rationale: Propylene glycol is the carrier solvent used to administer intravenous diazepam and lorazepam. Toxicity is rare but can occur with dosing above the recommended range and/or renal impairment, and presents as hyperosmolarity and anion gap metabolic acidosis, which progresses to multiple- organ failure. Midazolam does not use the same solvent so is not effected. Treatment consists of stopping the offending agent, with dialysis sometimes required (choice D is correct).

A 3-year-old is transferred from a small hospital after ingestion of a strong alkali industrial cleaning product. The child is stable but refusing to take orals. Endoscopy reveals circumferential grade 2B esophageal burns. Which of the following treatments should be considered? A. Esophageal stenting B. Lifelong proton pump inhibitor treatment C. Sclerotherapy of lesions D. Systemic corticosteroids

The correct answer is D. Rationale: Several studies have suggested systemic corticosteroids decrease stricture formation for 2B injuries. Esophageal stenting may be helpful for management of a stricture but is not indicated at this time (choice A in incorrect). Sclerotherapy is nonindicated (choice C is incorrect), and acid suppression after healing is not required (choice B is incorrect).

A 42-year-old woman is admitted to an intensive care unit after an intentional β-blocker overdose. An electrocardiogram shows QTc 600 ms and QRS 91 ms. Her heart rate is 60/min and blood pressure is 100/70 mm Hg when she suddenly develops torsades de pointes. Which one of the following β-blocker is most likely involved in this overdose? A. Atenolol B. Carvedilol C. Labetaolol D. Sotalol

The correct answer is D. Rationale: Sotalol is a potassium channel blocker (Vaughan-Williams class III), which means it can cause QTc prolongation and subsequent torsades de pointes (choice D is correct). Atenolol is not known to prolong the QTc (choice A is incorrect), and labetolol and carvedilol are both β- and α-adrenergic antagonists (choices B and C are incorrect).

Three coworkers present to the emergency department with progressive generalized weakness, severe myoclonus, fasciculations, disorientation, and difficulty speaking. They work together at a fruit and produce warehouse where the illness started 2 days ago. They vary in age from 22 to 33 years, have no past medical history, and take no medications. On physical examination, they are alert but mildly confused, with vital signs only significant for a mild sinus tachycardia. Lung, cardiac, and abdominal examinations are unremarkable. Neurologic examination confirms the findings upon presentation. Finger-stick blood glucose and venous carboxyhemoglobin levels are normal, but in one worker the serum chloride is 112 mEq/L (normal 96 mEq/L to 106 mEq/L). Which one of the following is the most likely explanation for this clinical presentation? A. Carbon monoxide toxicity B. Ethanol toxicity C. Hepatic encephalopathy D. Methyl b

The correct answer is D. Rationale: The acute presentation of new clinical manifestations in a group of people from a single location suggests an exposure to a toxicologic agent. The central nervous system disorder, myoclonus, mildly elevated serum chloride concentration, and history of work at a fruit and produce warehouse indicate methyl bromide toxicity (choice D is correct). Methyl bromide is a fumigant, and the common route of exposure is inhalation. The carboxyhemoglobin concentration was unremarkable on the venous blood gas (choice A is incorrect), ethanol toxicity does not cause hyperchloremia (choice B is incorrect), and the time of onset for disease in these workers was not consistent for hepatic encephalopathy (choice C is incorrect).

What is the goal of naloxone treatment in a patient who overdosed on an opioid? A. Normal mental status B. Normothermia C. Reversing miosis D. Spontaneous ventilation

The correct answer is D. Rationale: The goal of naloxone treatment is to reverse the ventilatory depressant effects of an opioid overdose and allow the patient to resume adequate spontaneous ventilation (choice D is correct). Intravenous naloxone dosing (starting with 0.04 mg) should be titrated to this desired clinical effect. Waking the patient up completely may precipitate acute opioid withdrawal leading to significant distress for both patient and healthcare staff (choice A is incorrect). Pupil size is not a reliable or useful measure of naloxone effect (choice C is incorrect). Clinicians should implement guideline recommended measures, and not rely on naloxone effects, for hypothermic patients (choice B is incorrect).

You are managing a patient with refractory seizures when additional history becomes available from questioning family members. After nonadherence to her tuberculosis medications, the patient attempted to "catch up" by taking 18 of her 300 mg isoniazid tablets all at once. Which of the following intravenous medications would be the most appropriate to administer at this time? A. Levetiracetam 40 to 60 mg/kg B. Lorazepam 4 mg C. Phenobarbital 20 mg/kg D. Pyridoxine 5.4 g

The correct answer is D. Rationale: The mechanism of toxicity of isoniazid is an absolute decrease in γ-aminobutyric acid (GABA) production. The most specific treatment for isoniazid overdose is a gram-per-gram dose of pyridoxine to increase GABA formation (choice D is correct). All of the other listed medications work by potentiating GABAA receptor activity, which means they require the presence of GABA and would therefore be ineffective in this case.

Which one of the following statements regarding paraquat poisoning is correct? A. Early lung transplant (within 3 weeks) is likely B. Hemodialysis is recommended C. Only Fuller's earth should be used for gastrointestinal decontamination D. Oxygen therapy may increase pulmonary toxicity

The correct answer is D. Rationale: The pathophysiology of paraquat toxicity is secondary to the formation of paraquat free radical from NADPH reduction which subsequently reacts with oxygen to form superoxide free radicals. The superoxide free radicals lead to the depletion of NADPH, lipid peroxidation, and apoptosis. Unlike diquat, paraquat is actively transported in the alveolar cells, causing much more pulmonary toxicity. Excess oxygen increases the formation of superoxide free radicals (choice D is correct). Oxygen supplementation should be limited to maintaining oxygen saturation no higher than 90%. Lung transplantation within the first few weeks after paraquat exposure may not be possible due to residual paraquat concentration in the blood (choice A is incorrect). The measurement of serum paraquat may be helpful for determining the timing of lung transplantation. Hemodialysis is not useful for preventing paraquat toxicity (choice B is incorrect). Either Fuller's earth or activated charcoal can be used to bind to paraquat in the gastrointestinal tract (choice C is incorrect). Because the prognosis for paraquat ingestion is so grave, early gastrointestinal decontamination with either of them is critical.

A 27-year-old male patient presents to the emergency department complaining of petechiae on his legs. He is otherwise asymptomatic. The following laboratory results are obtained: Patient Result Reference Interval Hemoglobin 8.0 g/dL Hematocrit 24% WBC 7.0 K/μL Platelet count 8 K/μL LDH 794 RBC antibody screen Panreactivity Negative DAT (Coombs test): IgG Strong positive Negative DAT (Coombs test): C3 Negative Negative What is the most appropriate management at this time? A. Platelet transfusion B. Platelet and red cell transfusion C. Therapeutic plasma exchange (TPE) D. No transfusion and no TPE

The correct answer is D. Rationale: The patient's likely diagnosis is Evans syndrome, a combination of autoimmune hemolytic anemia and autoimmune destruction of platelets. Other than the petechiae on his legs, he has no signs or symptoms of anemia or thrombocytopenia. Thus, red cell and platelet transfusions are not indicated (choices A and B are incorrect). In patients with Evans syndrome and/or ITP, platelet transfusion is only indicated if the patient is experiencing a critical bleeding episode. TPE is the treatment of choice for thrombotic thrombocytopenia purpura (TTP) but is not indicated for a patient with Evans syndrome (choice C is incorrect). First-line treatment for Evans syndrome is usually corticosteroids or intravenous immunoglobulin (IVIG).

A 35-year-old man ingested two tablets of aluminum phosphide 3 hours prior to arrival to the hospital. He vomited at home. He is currently unresponsive and intubated. Activated charcoal was administered via nasogastric tube. His blood pressure is 90/50 mm Hg, with heart rate 120 beats per minute. His chest radiograph demonstrates diffuse bilateral infiltrates. His electrocardiogram shows sinus rhythm at 120 beats per minute, with frequent premature ventricular beats, and nonspecific T-wave abnormalities. Bedside point of care echocardiogram indicates diffuse hypokinesis with low ejection fraction. Arterial blood gas on 80% oxygen is pH of 7.15, PCO2 of 42 mm Hg, and PO2 of 80 mm Hg. Serum electrolytes are remarkable for a bicarbonate of 14 mEq/L and anion gap of 21 mEq/L. Which of the following interventions is most likely to reduce this patient's mortality? A. Hemodialysis B. High-dose diazepam and epinephrine inf

The correct answer is D. Rationale: There are no randomized studies on the use of extracorporeal membrane oxygenation (EMCO) for phosphine poisoning. However, an observational study in a cohort of severely poisoned patients demonstrated significantly lower mortality rates (33%) with ECMO therapy compared to conventional therapy (87%) (choice D is correct). Hemodialysis does not remove phosphine and has no significant impact on patient outcomes (choice A is incorrect). Mechanical ventilation, high-dose diazepam, and epinephrine infusion are considered the primary therapy for chloroquine overdose, not phosphine (choice B is incorrect). N-acetylcysteine is the standard therapy for acetaminophen toxicity and has limited adverse effects, with potential benefits from animal models for phosphine toxicity as a free radical scavenger. However, there is inadequate human data on the use of intravenous N-acetylcysteine for phosphine toxicity (choice C is not the best answer). Not listed here as a choice, magnesium would also be incorrect; after initial reports on the potential benefits, subsequent studies did not confirm its utility.

A 35-year-old man is brought in by emergency medical services because he is extremely violent and agitated after ingesting phencyclidine (PCP). A paramedic administered a total of 20 mg of midazolam intravenously in the field without effect. Vital signs on arrival are blood pressure 200/110 mm Hg, pulse 118/minute, respiratory rate 36/minute, and temperature 40 °C, with SpO2 92% on room air and a finger stick blood glucose of 200 mg/dL. You administer a total of 40 mg of midazolam and 20 mg of haloperidol intravenously without effect. The patient continues to violently pull on his restraints. What is the most appropriate next step in management? A. Initiate dexmedetomidine infusion and forced diuresis B. Initiate ketamine infusion and urine alkalinization C. Initiate propofol and chlorpromazine infusions D. Initiate rapid sequence intubation and active cooling

The correct answer is D. Rationale: This patient failed multiple sedative agents and is persistently agitated and violent with severely abnormal vital signs. He needs to be emergently paralyzed, sedated, and cooled (choice D is correct). Of note, he should be paralyzed with a nondepolarizing paralytic like rocuronium because of the risk of hyperkalemia related to rhabdomyolysis. Succinylcholine should be avoided. Dexmedetomidine, while often used as sedative agent, is not a first-line agent for the treatment of PCP toxicity like benzodiazepines. Furthermore, urine acidification, alkalinization, and forced diuresis are unproven treatments for phencyclidine (PCP) intoxication (choices A and B are incorrect). Initiation of propofol and chlorpromazine infusions is unlikely to help as has already failed antipsychotics, and a propofol infusion should not be used in a patient without a secured airway (choice C is incorrect).

A 37-year-old man complains of neck and back spasms 3 hours after being bitten by a black widow spider on his right leg. He has a past medical history of asthma treated with albuterol and fluticasone inhalers. Vitals are blood pressure 159/74 mm Hg, heart rate 103 beats/min, and respiratory rate 22 breaths/min. He receives one 30 mg dose of ketorolac, 1 mg of hydromorphone, and 1 mg of lorazepam but continues to have painful neck and back spasms. What is the most appropriate next step in management? A. Black widow spider antivenom 1 vial B Diphenhydramine 50 mg C. Indomethacin 50 mg D. Lorazepam 2 mg every 30 minutes

The correct answer is D. Rationale: This patient has minimal risk of sequelae from the mild tachycardia and hypertension, and so there is time to titrate up standard therapy (opioids and benzodiazepines). Additional doses will be needed of both, but the lorazepam is certainly underdosed at this point (choice D is correct). Antivenom is a whole-immunoglobulin equine antivenom with a significant rate of anaphylaxis. This case resulted in death from cardiac arrest when antivenom was given.

A 23-year-old man is brought to the emergency department after being found obtunded on the street. The history suggests that he may have ingested 180 mg of his friend's methadone during a suicide attempt. On arrival he has a Glasgow Coma Scale (GCS) score of 6, with hypoactive bowel sounds and pinpoint pupils. His initial vital signs are temperature 36C, heart rate 56/min, blood pressure 103/70 mm Hg, respiratory rate 6, and oxygen saturation 84% on room air. Blood glucose is 4.6 mmol/L. Initially, his ventilation was supported through bag-valve-mask (BVM) apparatus but after three doses of 0.4 mg IV naloxone there is improvement in his respiratory effort. He is transferred to the ICU. What is the most appropriate management? A. Intubation for airway protection B. Naloxone boluses as needed plus activated charcoal C. Naloxone infusion at 0.4 mg/h D. Naloxone infusion at 0.8 mg/h

The correct answer is D. Rationale: This patient's presentation is consistent with opioid toxicity. Immediate resuscitation and administration of antidotal therapy are the most important initial steps in management. Given the hypoxia and bradypnea, supplemental oxygen without assisted ventilation is of minimal benefit. As a disease of hypoventilation, opioid toxicity requires ventilation. This is achieved through BVM and administration of opioid antagonists. Methadone is a long- acting opioid, with expected clinical toxicity to last beyond 24 hours. The proper dosing of a naloxone infusion per hour is 2/3 the dose required to adequately reverse the patient. This patient responded to 1.2 mg IV naloxone, so the infusion dose should start at 0.8 mg/h (choice D is correct). If naloxone was not available, intubation would be an option, but there is no need to do so at this time (choice A is incorrect). Methadone is a liquid product with rapid absorption, so activated charcoal would be of no benefit and potentially harmful as the fluctuating level of consciousness may result in aspiration, and lower doses of naloxone would be insufficient (choices B and C are incorrect).

A 38-year-old man is admitted to the ICU after being intubated for severe agitation. He was brought to the emergency department after being detained at an international airport for aggressive behavior. He required prehospital sedation with intramuscular midazolam to facilitate transport. On arrival, his vital signs were temperature 40.2 C, HR 165/min, BP 180/100 mm Hg, respiratory rate 24, and oxygen saturation 96% on room air. Blood glucose 10.5 mmol/L (189 mg/dL). He was intubated using rocuronium and was started on a propofol infusion at 4 mg/kg/hr. Vitals signs have not significantly changed. What is the most appropriate management? A. Active cooling B. Active cooling, increased sedation C. Active cooling, increased sedation, and β-blockade D. Active cooling, increased sedation, and paralysis

The correct answer is D. Rationale: This patient's presentation is consistent with significant toxicity from sympathomimetic agents (such as cocaine or amphetamines). His most immediate life-threatening problem is his hyperthermia, and goal is to cool the patient, which will be facilitated by a neuromuscular blocker (choice D is correct). Cooling with sedation alone are insufficient (choices A and B are incorrect) and β-Blockade is not indicated (choice C is incorrect). Sympathomimetic toxicity is a centrally driven disease (symptoms are due to an increase in excitation in the central nervous system). β-Blockade does not address this, as it only targets the peripheral symptoms of that centrally mediated process. Furthermore, β-blockers pharmacologically result in unopposed alpha stimulation, though the clinical significance of this has been recently debated.

A 20-year-old woman has taken an unknown medication during a suicide attempt. She presents complaining of chest pain with her heart racing and pounding. Her vital signs are blood pressure 150/90, heart rate 130 bpm, respiratory rate 18 breaths/min, and temperature 100.4 °F. Her physical examination is notable for anxious appearance with tremulousness, but her mental status is oriented to person, place, and time. Her pupils are dilated symmetrically. Mucous membranes are moist. Cardiac examination is notable for a rapid rate without murmurs, lungs are clear, abdomen is nondistended with active bowel sounds, and neurological exam shows a slight tremor and hyperreflexia but no ankle clonus. Which of the following medications did this patient most likely overdose? A. Dextromethorphan B. Diphenhydramine C. Donepezil D. Pseudoephedrine

The correct answer is D. Rationale: This presentation is consistent with a sympathomimetic toxidrome (tachycardia, hypertension, anxiety, mydriasis, and tremor), making pseudoephedrine the right answer (choice D is correct). Dextromethorphan enhances central serotonergic tone by blocking its reuptake as well as increasing its release and is an N-methyl-D-aspartate receptor antagonist that is associated with postsynaptic 5-HT stimulation. Dextromethorphan administrated at supratherapeutic doses or in combination with serotonergic agents could induce a serotonin syndrome that also has overlapping signs and symptoms of anticholinergic toxicity including hypertension, tachycardia, fever, and agitated delirium. The major distinguishing feature is often muscle rigidity and sustained clonus (choice A is incorrect). Diphenhydramine would produce an anticholinergic syndrome (choice B is incorrect). There are many overlapping features with the sympathomimetic and anticholinergic syndromes. They both may cause tachycardia, hypertension, temperature elevation, mydriasis, and seizures and motor signs with tremor common in the sympathomimetic syndrome and myoclonus in the anticholinergic syndrome. The major distinguishing features for anticholinergic toxicity are altered mental status changes, that may include agitated delirium and garbled speech, and dry mucous membranes with diminished bowel sounds. Donepezil would produce a cholinergic syndrome with bradycardia and hypotension (choice C is incorrect).


Set pelajaran terkait

Econ 330 Exam 2 Practice Questions

View Set

Institutions of government part 2

View Set

AWS Academy Cloud Architecting [2606] - Module 14 Knowledge Check

View Set

SOS Language Arts 800: Unit 9- Etymology of English Words

View Set

anatomy 4: cranial nerves function & innervation

View Set

Lyme Disease: Borrelia burgdorferi

View Set

Maternity Chpt. 4 Common Reproductive Issues 5-8

View Set